Sunteți pe pagina 1din 75

ACCA F9 Financial management notes

Exam
A total of 4 questions, each worth 25 marks = 100 marks.
About this note
This note is created by focusing on the study guide of F9 found in ACCA website, satisfying the
requirements in study guide as much as possible and covering what you need to know to pass this
exam. The notes only covered main ideas, make sure you practice past year questions to be fully
prepared. It is good if you can bring forward your knowledge from previous studies to understand
better, especially F5 and it gives you advantage if you have FFM knowledge.

Syllabus areas

Page numbers

A: Financial management function

2-4

B: Financial management environment

5-7

C: Working capital management

8-19

D: Investment appraisal

20-34

E: Business finance

35-45

F: Cost of capital

46-56

G: Business valuations

57-63

H: Risk management

64-75

Financial management function


1. Nature and purpose of financial management
Financial management is the management of activities associated with the efficient acquisition and use
of short and long-term financial resources, to ensure the objectives of the company are achieved.
There are three key decisions in financial management:
1. Investment this includes investment in projects (long-term) and working capital (short-term).
2. Financing company will decide the best balance of equity and debt.
3. Dividend how much and how frequently dividend will be paid.
Relationship between financial management and financial and management accounting
Financial management relies on financial accounting and management accounting. For example, the
information in financial statement and other past information generated from financial accounting
could assist in future financial planning. Financial planning is further assisted by management
accounting, for example relevant cash flows are identified. Also, management accounting plays a role
in financial control, ensuring objectives are being met or assets being used efficiently.
2. Financial objectives and the relationship with corporate strategy
Corporate objectives and strategy
Corporate objectives are relevant for the organisation as a whole, relating to key factors for business
success (critical success factors or CSFs). The primary financial objective of profit making organisation is
to maximise shareholder wealth, other financial objectives include profit maximisation, earnings per
share (EPS) growth, market share growth etc. Strategy is a course of action to achieve an objective so
corporate strategy depends on corporate objectives.
3. Stakeholders and impact on corporate objectives
Stakeholders and their objectives
Stakeholders are groups or individuals having a legitimate interest in the activities of an entity. There
are three types of stakeholders and each having their specific objectives:
1. Internal stakeholders employees and management.
2. Connected stakeholders shareholders, customers, suppliers and lenders.
3. External stakeholders community, government and pressure groups.
Examples of stakeholders objectives:
1. Shareholders want to maximise their wealth.
2. Trade payables want to be paid full amount at due date.
3. Management wants to maximise their rewards.
4. Government wants sustained economic growth and high levels of employment.

Conflict between stakeholder objectives and agency theory


Sometime conflict will arise between stakeholder objectives, for example shareholders may encourage
management to pursue risky strategies in order to maximise returns (high risk, high return), but finance
providers prefer lower risk policies. Management has to try and balance the interest of different
stakeholders.
According to agency theory, directors/managers are agents of the shareholders being tasked to run the
entity in the best interest of shareholders. However conflict may also arise between directors personal
objectives (eg. increase personal remuneration) and stakeholders objectives. If directors fail to act in
the best interests of shareholders, this leads to sub-optimal returns to shareholders. This potential lost
of wealth for shareholders is known as agency costs. Agency costs can also be defined as costs
incurred by the principal to monitor the agent.
Measuring achievement of corporate objectives
Ratio analysis
1. Return on capital employed (ROCE) = profit before interest and tax (PBIT)/(equity + debt) x 100%.
2. Return on equity = profit available to ordinary shareholders/equity x 100%.
3. EPS = profit available to ordinary shareholders/number of shares.
4. Dividend yield = Dividend per share/market price per share x 100% (indicates the return on capital
investment relative to price).
5. Dividend cover = EPS/dividend per share (measures the ability to maintain existing level of dividend).
6. Price earnings ratio (P/E ratio) = market price per share/EPS (reflect shareholders expectation of
companys future performance).
It is useful to bring forward your knowledge of ratio analysis to this paper (if you studied F5 or F7).
Total shareholder return (TSR)
TSR = (Year-end share price share price at start of the year + dividend per share)/share price at start
of the year x 100% (total return to shareholders through dividend and capital gains).
Ways to encourage the achievement of stakeholder objectives
Firstly, we have to make sure that goal congruence (managers goals = organizations goals) is achieved,
probably by managerial reward schemes including performance-related pay, rewarding managers with
shares and share options schemes (this enables managers to buy an amount shares at fixed price which
is usually todays price, therefore managers are encouraged to take decisions that maximise future
share prices), these can be given to managers if target is achieved.
Regulatory requirements can also help:
1. Corporate governance (system by which companies are directed and controlled) the elements
include risk management, internal controls, accountability to stakeholders, conducting business in an
ethical and effective way.
2. Stock exchange listing regulations (rules and regulations to ensure stock market operates fairly and
efficiently).
3

4. Financial and other objectives in not-for-profit organisations


Not-for-profit organisations (NFPOs) are those organisations that are not formed primarily to make
profit but probably to serve public interest.
Objectives of NFPOs
Their objectives are mainly non-financial, which may relate to a level of service and difficult to quantify.
NFPOs have multiple objectives which are difficult to define. Therefore, it is later identified that value
for money (VFM) concept is very relevant to them, this means providing a service in a way which is
economical, efficient and effective.
VFM as objectives
NFPOs can take VFM as their objectives, this means achieving economy, efficiency and effectiveness,
the 3Es.
1. Economy can be achieved if company is able to obtain appropriate quantity and quality of inputs at
lowest cost. In other word, comparing inputs with money spent.
2. Efficiency can be achieved if company is able to get as much outputs as possible from the inputs. In
other word, comparing outputs with inputs.
3. Effectiveness can be achieved if companys outputs are in line with the objectives set. In other word,
comparing objectives with outputs.
Taking a school as an example, let say the objective is to increase the overall passing rate, the school is
said to be economical if teachers are trained with lowest possible cost, the school is said to be efficient
if the number of students passing the exam have increased with the trained teachers, the school is said
to be effective if there are 95% of students passing the exam which achieved the objective to increase
overall passing rate.
We can also say that economy + efficiency = effectiveness.
Measuring achievement of NFPO objectives
Other than looking at the VFM, customer satisfaction, competitive position, market share, resource
utilisation measures, benchmarking and so on could be useful as indicators of whether the company is
achieving the objectives.

Financial management environment


1. The economic environment for business
Macroeconomic policy targets
Macroeconomic policy is a government policy aimed to manage the economy by influencing the
performance and behaviour of the economy as a whole. The main targets include:
1. Full employment.
2. Price stability control inflation.
3. Balance of payments balance between imports and exports.
4. Economic growth improve living standards.
5. Acceptable distribution of income and wealth.
Components of macroeconomic policy
1. Fiscal policy action by government to spend money or collect money in taxes, to influence the
condition of the national economy. It can be used to manage aggregate demand (eg. raise tax to
reduce demand in the economy, spend more to increase demand in the economy).
2. Monetary policy aim to influence quantity of money, price of money (interest rate) and money
supply (total stock of money) in the economy.
3. Exchange rate policy control the value of the currency to change the prices of imports and exports.
Effects of government economic policy
If tax rises, it damages the companys profits and company may be reluctant to invest.
If interest rate rises, demand for money will reduce and inflation will fell, companys profits will
also fell.
If exchange rate rises (ie. own countrys currency strengthen), it increases export prices (higher
revenue) and lower cost of imports (cheaper purchase).
Government intervention in the economy
Government may intervene in the operation of free market when monopolies, mergers or restrictive
practices operate against public interest, when the free market fails to the amount of capital required
etc. Government will intervene through:
1. Competition policy to reduce a companys domination of a market (ie. monopoly) and to increase
the efficiency of the economy by stimulating competition.
2. Government assistance for business through official aid schemes (eg. giving grants, tax incentives)
or enterprise initiatives to encourage the formation of new businesses.
3. Green policies increase production costs as companies are required by legislation to reduce
environmental impact of their business operations. Green policies can be seen as leading to fairer
prices in particular product markets since these prices reflect more completely the economic resources
consumed in the production of the products offered for sale.
4. Corporate governance regulation.

2. The nature and role of financial markets and institutions


Financial intermediaries
Financial intermediaries bring together lenders and borrowers of money, either as broker (an agent
handling a transaction on behalf of others) or as principal (holding money balances of lenders for
lending on to borrowers). Examples of financial intermediaries are:
(i) Bank.
(ii) Building societies give loans to borrowers for house purchase.
(iii) Finance houses provide hire purchase service.
(iv) Insurance companies use policyholders premiums to invest.
(v) Pension funds collect contributions to invest for pensions.
(vi) Unit trusts raise funds by trading in shares and bonds.
(vii) Investment trust companies similar to unit trusts but trade in larger amount.
Benefits of financial intermediation are:
(i) Aggregation bank can aggregate the amount of money from lenders and then lend to borrowers
who need the money, this makes things easier for lenders and borrowers to lend or obtain money.
(ii) Risk reduction bank should be better at assessing credit risk.
(iii) Maturity transformation lenders may want to keep money for liquidity while borrowers may need
loan (long-term borrowing), financial intermediary can facilitate short-term and long-term needs of
lenders and borrowers, this is called maturity transformation.
Financial markets
These are the places where those requiring finance (Deficit Units) can meet those who are able to
supply finance (Surplus Units). There are two types of financial markets:
1. Money markets markets for short-term borrowing and lending, in wholesale amount. Money
markets include a primary market and a secondary market. The primary market is used by the central
bank and other approved banks and securities firms. The central bank uses it to balance shortages and
surpluses of cash. Main money markets financial instruments are:
(i) Deposits deposits of money in financial intermediaries.
(ii) Bills short-term financial assets which can be converted into cash at very short notice, by selling
them in the discount market.
(iii) Commercial paper short-term IOUs issued by large companies which can be held until maturity or
sold to others. It is issued when company wants to raise short-term money.
(iv) Certificates of deposits (CDs) fixed terms deposit, customer can obtain cash before the term is up
by selling CD in CD market.
Other secondary UK markets include:
(i) Local authority markets provide local authority bonds and bills.
(ii) Inter bank market unsecured loans between banks.
(iii) CDs market
(iv) Inter company market companies with surplus funds lend directly (through a broker) to those
which need to borrow. They do not involve financial intermediation and this is called disintermediation.
6

(v) Commercial paper market


(vi) Eurocurrency markets eurocurrency deposit is a foreign currency deposit, a deposit of own
countrys funds in other countries which have different currency, eg. deposit of US dollars with a bank
in London. This is an international money market and only suitable for larger companies.
2. Capital markets markets for trading in longer-dated securities such as shares and bonds. Examples
of capital markets include Stock Exchange (or stock market, for buying and selling of shares) and bond
market (for trading of debt securities). There is also an international capital market which trades
Eurobond (bond issued in foreign currency) which is also suitable for larger companies only.
Functions of stock market
In UK, the stock market is known as the London Stock Exchange. There are actually two markets within
this stock exchange. The first of these is the Official List. This is the top tier (level) of the market and is
only available for large companies who can meet the strict listing requirements. The second tier is the
Alternative Investment Market (AIM). The listing requirements for this market are less strict, hence it is
used by new and smaller companies. There are two main functions:
1. As primary markets they enable companies to raise new finance by issuing shares or bonds.
2. As secondary markets they enable existing investors to sell their investments.
Apart from these two, stock markets have two other important functions:
1. The owners of the company can sell some of their shares to new investors when the company comes
to the stock market for the first time, eg. just became public company.
2. Company can take over another company by issuing shares to finance the takeover, ie. share
exchange.
Risk/return trade-off
The concept of this is that investors in riskier assets expect to be compensated higher return. The risk
level of different securities can be ranked as follow, first being least risky:
1. Government bonds.
2. Company bonds/loan note usually secured against companys assets.
3. Preference shares rank behind debt in the event of liquidation.
4. Ordinary shares in the event of liquidation, ordinary shareholders are the last to be paid.

Working capital management


1. The nature, elements and importance of working capital
Working capital is the capital available for conducting the day-to-day operations of an organisation.
Net working capital can be calculated as current assets less current liabilities. Company needs working
capital to keep the trading activities on-going. Inventory, receivables, payables and cash are the major
elements of working capital.
Objectives of working capital management
Working capital management has two main objectives:
1. To ensure company has sufficient liquid resources to continue in business (minimise risk of
insolvency).
2. To increase profitability (maximise return on assets).
These two objectives often conflict as liquid assets give the lowest returns. For example, holding high
level of cash will improve liquidity position but will also harm profits because if the cash was used more
profits could be made.
Role of working capital management
Working capital management aims to balance not having too much or too less working capital.
Working capital management involves:
Controlling the liquidity position.
Controlling the working capital elements which are inventory, receivables and payables.
Cash is the most liquid asset, inventory is considered non-cash and so it is least liquid asset.
Receivables fall in the middle of cash and inventory.
2. Management of inventories, accounts receivable, accounts payable and cash
Cash operating cycle
Cash operating cycle/working capital cycle is the period between the suppliers being paid and the cash
being received from the customers. Working capital cycle in a manufacturing business equals:
The average time that raw materials remain in stock (inventory days)
- period of credit taken from suppliers (payables days)
+ time taken to produce the goods (inventory days)
+ time finished goods remain in stock after production is completed (inventory days)
+ time taken by customers to pay for the goods (receivables days)
In brief, working capital cycle = inventory days + receivables days payable days.
Liquidity ratios
Liquidity ratios may help to indicate whether a company is over-capitalised, with excessive working
capital, or if a business is likely to fail. A business which is trying to do too much too quickly with too
little long-term capital is overtrading.
1. Current ratio = current assets/current liabilities, ideal is 2:1.
8

2. Quick or acid test ratio = (current assets inventories)/current liabilities, ideally should be at least
1:1.
3. Average collection period/receivables days = receivables/credit sales x 365 days, this shows the
length of time it takes for companys customers to pay. This formula can be changed to receivables
days/365 days x credit sales to get receivables.
4. Average payable period/payables days = payables/credit purchases x 365 days, this shows the time
taken for company to pay suppliers. This formula can be changed to payables days/365 days x credit
purchases to get payables.
5. Inventory turnover period (finished goods) = inventory/cost of sales x 365 days.
6. Raw materials days = raw materials inventory/purchases x 365 days.
7. Work-in-progress (WIP) period = (WIP inventory/cost of sales x % of completion) x 365 days.
8. Inventory turnover ratio = cost of sales/average inventory.
9. Sales revenue/net working capital ratio = sales/(current assets current liabilities), this shows the
level of working capital supporting sales and working capital must increase in line with sales to avoid
liquidity problems.
Symptoms of overtrading are increased revenue, increased current/non-current assets, current
liabilities more than current assets, assets financed by credit and not share capital, reduced current
and quick ratios, inventory and receivables are more than sales.
Managing inventory
Important techniques include economic order quantity (EOQ) and just-in-time (JIT). It is very useful to
bring forward your knowledge from previous studies.
EOQ is the optimal ordering quantity for an item of inventory that will minimise costs, at the same time
balancing the need to meet customer demand. Inventory costs include:
(i) Holding costs eg. rental of warehouse, theft of stock.
(ii) Ordering costs eg. telephone charges, delivery costs.
(iii) Shortage costs eg. loss of sale
(iv) Purchase costs price of the goods
EOQ or Q =

2cd/h, c = cost of per order for one year, d = annual demand, h = holding cost per unit

of inventory for one year, Q = reorder quantity. (EOQ formula is given in exam).
Holding cost = Qh/2, ordering cost = cd/Q
Total annual cost = holding cost + ordering cost + purchase cost
Assumptions of EOQ formula are purchase costs are constant, lead time is constant, demand is
constant and no inflation.
When there are bulk discounts from other supplier and you have to decide whether to order based on
EOQ or take the bulk discounts, compare the total annual cost if used EOQ and the total annual cost if
takes the bulk discounts (company may order more to get the discount), the lower costs will be chosen.
Other formulas include:

Reorder level = maximum usage x maximum lead time, measure the inventory level at which
replenishment order should be placed.
Maximum level = reorder level + reorder quantity (minimum usage x minimum lead time), inventory
level should not exceed this level.
Minimum level/buffer inventory = reorder level (average usage x average lead time), inventory level
should not fall under this level.
Average inventory = reorder quantity/2 + minimum level
Just-in-time (JIT) aims to hold as little inventory as possible and production systems need to be very
efficient to achieve this. Deliveries will be small and frequent rather than in bulk. Company needs to
have a reliable supplier as that supplier will guarantee to deliver raw materials components of
appropriate quality always on time. Unit purchasing prices may be higher as supplier guarantees the
quality and also on time delivery. Workforce must also be flexible and multi-skilled in order to minimize
delay and eliminate poor quality production. Reduced inventory levels mean that a lower level of
investment in working capital will be required.
Managing receivables
Managing accounts receivable involves many aspects, mainly relating to offering credit and collecting
back the money. Credit control policies are guideline on giving credit, can be set based on before
offering credit (assess creditworthiness, check past record of customers), during credit period (monitor
the receivables) and after credit period (chase slow payers, aged receivables analysis). The amount of
total credit that a business offers depends on:
(i) The firms working capital needs and the investment in receivables.
(ii) Management responsibility for carrying out the credit control policy.
An important aspect of the credit control policy is to devise suitable payment terms, covering when
and how should payment be made.
Assessing creditworthiness
A credit assessment is a judgement about the creditworthiness of a customer. It provides a basis for a
decision as to whether credit should be granted. If the credit risk (possibility that the debt goes bad) is
high, the customers need to be managed carefully. The methods include:
1. Bank reference banks are cautious as they owe duties of care to customer and enquirer, therefore
the information about customers are limited.
2. Trade reference - get information from customers suppliers, useful but careful as customers will
give name of suppliers that they have good history with.
3. Check the credit ratings from the credit rating agency.
4. Ratio analysis.
5. Customer visits.
6. Press comments.

10

Monitoring accounts receivables


Aged receivables analysis and credit utilisation report can be used to monitor accounts receivable.
Example of aged receivables analysis is as follow:
Customer name
0-30 days
31-60 days
61-90 days
>90 days
Balance
ABC
1000
300
700
0
0
DEF
2000
0
900
200
900
Total
3000
300
1600
200
900
Example of credit utilisation report is as follow:
Customer name
Limit
Utilisation
%
ABC
300
280
93
DEF
100
50
50
This shows that ABC needs more credit and DEF does not.
The decision to extend credit given to customers will depend on factors such as:
1. Profits from extra sales.
2. Extra length of average debt collection period.
3. Required rate of return (cost of capital) on the investment in additional accounts receivable.
Example: Ice Co currently expects sales of $50000 per month and variable cost of sales is $40000 per
month (all payable in the month of sale). It is estimated that if credit period increased from 30 days to
60 days, sales volume will increase by 20%. If cost of capital is 10% per annum, decide whether to
extend credit period.
Solution: Remember that when sales volume increases, the variable cost of sales will also increase.
Contribution per month = 50000 40000 = $10000, new contribution = 10000 x 1.2 = $12000.
Extra contribution per month = 12000 10000 = $2000.
Accounts receivable for 60 days credit period (2 month) = $60000 (new sales) x 2 = $120000.
Accounts receivable for 30 days credit period (1 month) = $50000 x 1 = $50000.
Extra receivables = $120000 - $50000 = $70000.
Annual benefit from extending credit period = $2000 x 12 months $70000 x 10% = $17000.
Therefore, financially it is worthwhile to extend the credit period.
Collecting amounts owing
The benefits of action to collect debts must be greater than the costs incurred. To encourage
customers to pay on time, make sure that:
Customer is fully aware of the payment terms.
Invoice is correctly drawn up and issued promptly.
Awareness of customers system understand the payment system of customers business.
Queries are resolved quickly.
Monthly statements are issued promptly.

11

There should be efficiently organised procedures for ensuring that overdue debts and slow payers are
dealt with effectively, some examples are:
Issuing reminder letters
Chasing payment by telephone
Charge interest for late settlement
Employ services of debt collection agency (pay commission)
Send authorized person to visit and request payment
Take legal action
Offering early settlement discounts
Early settlement discounts may be offered to reduce average credit periods and investment in
accounts receivable which will also reduce interest costs. To decide whether to offer the discount,
compare cost of discount allowed and benefits from reduced accounts receivable.
Example: Fire Co has annual credit sales of $12000000 and three months are allowed for payment. The
company decides to offer 2% discount for payments made within ten days of invoice being sent and to
reduce the time allowed for payment to two months. It is estimated that 50% of customers will take
the discount. Company requires 20% return on investments and assume sales volume remain, decide
whether or not to offer the discount.
Solution: New accounts receivable = 10/365 x 50% x $12000000 + 2/12 x 50% x $12000000 = $1164384.
Current accounts receivable = 3/12 x $12000000 = $3000000.
Reduction in accounts receivable = 3000000 1164384 = $1835616.
Benefits of the reduced accounts receivable = $1835616 x 20% = $367123.
Cost of discount allowed = $12000000 x 50% x 2% = $120000.
Net benefit = 367123 120000 = $247123.
It is therefore financially beneficial to offer the early settlement discount.
The compound annual cost of early settlement discount can be calculated by (100/100 d) ^ (365/t)
1, d = discount, t = time difference between cash discount date and the credit term.
Example: A company offers its goods to customers on 30 days credit. It also offers a 2% discount if
payment is made within 10 days of the date of invoice, calculate compound annual cost of offering the
early settlement discount.
Solution: Compound annual cost = (100/98) ^ (365/20) 1 = 44.6%.
Alternatively, 2/98 is the cost of discount and to convert it to annual percentage:
(1 + 2/98) ^ (365/20) 1 = 44.6%.
Using factoring and invoice discounting
Some businesses might have difficulties in financing amounts owed by customers, they can employ the
service of factoring. Factoring is an arrangement to have debts collected by a factor company which
advances a proportion of the money that it is due to collect.
12

An easier way to understand factoring is through steps:


1. Company asks for factoring service from factor.
2. Factor will administer the sales ledger (control the receivables of the company) and give money in
advance to company (about 80%).
3. After factor received money from receivables, factor will pay back the company an amount which
has been deducted for interest.
There are two types of factoring:
(i) With recourse if debt cannot be collected, factor can claim back the advance from company
(ii) Without recourse if debt cannot be collected, factor cannot claim back the advance from
company.
To determine whether it is financially viable to use factoring service, compare cost of factoring and cost
of not factoring.
1. The cost of not factoring this means that the costs if company uses own system of managing
receivables. Examples include credit controller salaries, administration costs and interest charged on
overdraft.
2. The cost of factoring examples of costs are interest charged for advance of money, interest
charged for financing remaining receivables and administration fees.
Example: Rock Co makes annual credit sales of $1500000. Credit terms are 30 days, average collection
period has been 45 days with 0.5% of sales resulting in bad debts, administration costs are $30000. A
factor would charge annual fee of 2.5% of credit sales and payment period would be 30 days. The
factor would also provide an advance of 80% of invoiced debts at an interest rate of 14%. Interest on
overdraft is 13.5%. Decide whether to accept the factors service.
Solution: Accounts receivable without factoring = 45/365 x 1500000 = $184932.
Accounts receivable with factoring = 30/365 x 1500000 = $123288.
Cost of not factoring
Admin cost = $30000
Interest on overdraft = 13.5% x 184932 = $24966
Bad debt = 0.5% x 1500000 = $7500
Total cost = $62466
Cost of factoring (80% financed by factor and 20% financed by overdraft)
Interest on advance = $123288 x 80% x 14% = $13808
Interest on overdraft = $123288 x 20% x 13.5% = $3329
Admin fee = $1500000 x 2.5% = $37500
Total cost = $54637
It is financially viable for the company to use factoring service as the cost is lower. The net benefit of
using factor = $62466 - $54637 = $7829.
Invoice discounting is the purchase of trade debts at a discount. Invoice discounting enables company
to raise working capital. It is similar to factoring, but invoice discounter does not administer sales
ledger, with this, customers will not know that the company is employing this service.
13

Managing foreign accounts receivable


Company with foreign accounts receivable or exporters might have problems of larger inventories and
accounts receivable, and an increased risk of bad debts due to the transportation time and additional
paperwork involved in sending goods abroad. There are several ways for exporters to overcome these
problems:
1. Reduce investment in foreign accounts receivable.
2. Reducing the bad debt risk assessing creditworthiness is essential.
3. Export factoring the functions of overseas factor are the same as the factor discussed.
4. Documentary credits customer requests the bank of his country to issue letter of credit in favour of
exporter, then the issuing bank will guarantee payment to the beneficiary (exporter).
5. Countertrade goods are exchanged for other goods, it is a form of barter.
6. Export credit insurance insurance against the risk of non-payment by foreign customers.
Summary of accounts receivable management
There are four key areas of accounts receivable management told by examiner in the pilot paper:
1. Policy formulation establishing a framework within which management of accounts receivable in
an individual company takes place. The elements to be considered include establishing terms of trade,
such as period of credit offered and early settlement discounts: deciding whether to charge interest on
overdue accounts; determining procedures to be followed when granting credit to new customers;
establishing procedures to be followed when accounts become overdue, and so on.
2. Credit analysis assessment of creditworthiness depends on the analysis of information relating to
the new customer. This information is often generated by a third party and includes bank references,
trade references and credit reference agency reports. The depth of credit analysis depends on the
amount of credit being granted, as well as the possibility of repeat business.
3. Credit control once credit has been granted, it is important to review outstanding accounts on a
regular basis so overdue accounts can be identified. This can be done, for example, by an aged
receivables analysis. It is also important to ensure that administrative procedures are timely and
robust, for example sending out invoices and statements of account, communicating with customers
by telephone or e-mail, and maintaining account records.
4. Collection of amount due - Ideally, all customers will settle within the agreed terms of trade. If this
does not happen, a company needs to have in place agreed procedures for dealing with overdue
accounts. These could cover logged telephone calls, personal visits, charging interest on outstanding
amounts, refusing to grant further credit and, as a last resort, legal action. With any action, potential
benefit should always exceed expected cost.
Managing accounts payable
Effective management of payables involves seeking satisfactory credit terms from suppliers, getting
credit extended during periods of cash shortage, and maintaining good relations with suppliers.

14

Using trade credit effectively


Trade credit is a source of short-term finance because it helps to keep working capital down. It is
usually cheap as suppliers rarely charge interest. However, if the company attempts to make maximum
use of trade credit, potential cost might include loss of suppliers goodwill and loss of available early
settlement discount.
Evaluating the benefits of discounts for early settlement and bulk purchase
The percentage compound annual cost of not taking discount can be calculated by (100/100 d) ^
(365/t) 1, ie. same as the one used in accounts receivable. Now an example is taken from June 2011
question 4.
Example: ZPS Co places monthly orders with a supplier for 10,000 components that are used in its
manufacturing processes. Annual demand is 120,000 components. The current terms are payment in
full within 90 days, which ZPS Co meets, and the cost per component is $750. The cost of ordering is
$200 per order, while the cost of holding components in inventory is $100 per component per year.
The supplier has offered either a discount of 05% for payment in full within 30 days, or a discount of
36% on orders of 30,000 or more components. If the bulk purchase discount is taken, the cost of
holding components in inventory would increase to $220 per component per year due to the need for
a larger storage facility.
Assume that there are 365 days in the year and that ZPS Co can borrow short-term at 45% per year,
calculate if ZPS Co will benefit financially by accepting the offer of early settlement discount or bulk
purchase discount.
Solution:
Early settlement discount
Since we are given the interest rate, we can calculate the value of discount in % and compare.
Value of the discount expressed in annual percentage = (100/99.5)^(365/60) 1 = 3.1%.
Since the value of the discount is lower than the short-term borrowing rate, it is not financially
beneficial to accept the discount. (Note: Another way is to compare the value of discount in $ and the
cost, that is the increase in finance cost due to reduction in payables).
Bulk purchase discount
Compare the annual inventory cost of order at 10000 and at 30000 components.
Cost of order at 10000 components
Holding cost = 10000/2 x $1 = $5000
Ordering cost = 120000/10000 x $200 = $2400
Purchase cost = 120000 x $7.50 = $900000
Total cost = $907400
Cost of order at 30000 components
Purchase cost = 120000 x $7.50 x 96.4% = $867600
Holding cost = 30000/2 x $2.2 = $33000
Ordering cost = 120000/30000 x $200 = $800
15

Total cost = $901400


It is financially beneficial for ZPS Co to accept the bulk purchase discount as it saves $6000 (907400
901400) per year.
Managing foreign accounts payable
Foreign accounts payable are subject to exchange rate risk. Depreciation of a domestic currency will
cause the cost of supplies more expensive. The management of exchange rate risk will be discussed in
last syllabus area.
Managing cash
Reasons of holding cash
Keynes had identified three reasons why company should hold the surplus cash rather than investing it:
(i) Transaction motive hold cash to meet regular commitments.
(ii) Precautionary motive hold cash in case of emergency purpose.
(iii) Speculative motive hold cash to wait for good opportunity to invest (eg. when interest rates rise).
Preparing cash flow forecasts
A cash flow forecast is a detailed forecast of cash inflows and outflows. The timing is important, for
example a new delivery vehicle was brought in June and the cost of $8000 is to be paid in August, then
you should record $8000 in August. Remember, we include the item only when cash is receipt or paid.
A good step to prepare cash flow forecast is to set out the pro-forma first and include amount which
does not or just require easy calculation, then only do workings and include the rest of the amount.
Example of cash flow forecast format is as follow:
Cash flow forecast for six months ending 31 December 2011
Jul
Aug
Sep Oct
Nov Dec
Receipts
$
$
$
$
$
$
Credit sales
100
100
Payments
Corporation tax
50
Materials
10
60
Surplus/ (Deficit)
40
Balance b/f
10
50
Balance c/f
50
Treasury management
Treasury management in a modern enterprise covers various areas, and in larger business may be a
centralised function. The role of treasurer includes liquidity management, funding management,
currency management, formulating corporate financial objectives, handling corporate finance and risk
management. Advantages of centralised treasury management are: better short-term investment
16

opportunities, improved foreign exchange risk management, able to employ experts (cash is pooled)
and easier to manage cash.
Cash management models
Optimal cash holding levels can be calculated using Baumol model and Miller-Orr model.
Baumol model is based on the idea that deciding on optimum cash balances is similar to deciding on
optimum inventory levels. It is similar to EOQ, it is based on the formula Q =

, Q is cash to be

raised, F is fixed cost of obtaining new funds, S is amount of cash to be used, I is interest cost of holding
cash.
Example: Fire Co faces a fixed cost of $4000 to obtain new funds. There is a requirement for $24000 of
cash over each period of one year for the foreseeable future. The interest cost of new funds is 12% per
annum, interest rate earned on short-term securities is 9% per annum. How much finance should Fire
Co raise at a time?
Solution: Interest cost of holding cash = 12% - 9% = 3%.
Q=

= $80000. Q is like reorder quantity, so this means to raise $80000 every time.

The limitations of Baumol cash management model are as follow:


(i) In reality, amounts required over future periods will be difficult to predict with much certainty.
(ii) The model works satisfactorily for a firm which uses cash at steady rate but not if there are larger
inflows and outflows of cash over time.
(iii) There may be difficulty in predicting future interest rates.
Miller-Orr model is based on the idea that variance of cash flows, transaction costs and interest rates
will affect the return point (normal level of cash balance). Lower limit will be set, then upper limit and
return point depend on lower limit. The following formulas are given in exam:
Return point = Lower limit + (1/3 x spread)
Spread = 3 (3/4 x transaction cost x variance of cash flows/interest rate)^1/3
The upper limit is calculated as lower limit + spread as spread shows the fluctuation between lower
limit and upper limit. Note that if standard deviation is given instead of variance, remember to square
it to get variance. This model is applied to manage daily cash.
Example: Ice Co has the following data, formulate a decision rule using Miller-Orr Model:
1. Minimum cash balance is $8000. (This is the lower limit)
2. Variance of daily cash flows is 4000000, equivalent to standard deviation of $2000 per day.
3. Transaction cost for buying and selling securities is $50. Interest rate is 0.025% per day.
Solution: Spread = 3 x (3/4 x 50 x 4000000/0.00025)^1/3 = $25303, lets take $25300.
Upper limit = Lower limit + $25300 = $8000 + $25300 = $33300.
Return point = $8000 + 1/3 x 25300 = $16433, lets take $16400.

17

The decision rule is if cash balance reaches $33300, buy $16900 (33300 16400) of marketable
securities (reduce the cash balance to return point). If cash balance falls to $8000, sell $8400
marketable securities (increase the cash balance to return point).
Short-term investment of surplus funds
Before investing surplus funds, key factors to consider are as follow:
(i) Risk the higher the risk, the higher the return. There are two types of risks, systematic risk (risk
that affects the whole market and cannot be diversified away) and unsystematic risk (risks that affects
only specific market, can be reduce by diversification, means to hold more than one/portfolio of
investment).
(ii) Liquidity the ease of converting into cash, high liquid low return.
(iii) Maturity the duration of investment, long maturity high return.
(iv) Return after considered risk, liquidity and maturity, company is in position of considering how
much return they want.
The investment options available include:
1. Certificate of deposit (CD) a certificate indicating that a sum of money has been deposited with a
bank and will be repaid at a later date. As CDs can be bought and sold in the CD market any time, they
are liquid type of investment.
2. Treasury bills IOUs issued by government, promising to pay a certain amount to their holder on
maturity.
3. Shares.
4. Deposit in bank or similar financial institution.
5. Bonds.
3. Determining working capital needs and funding strategies
Working capital requirements
The amount of current assets and current liabilities can be determined by looking at the working
capital cycle components. You can change the formula of receivable, inventory and payable days to
find the amount.
Example: The annual cost of purchasing direct materials is $450000 and raw materials are in inventory
for three months.
Solution: Raw material days = raw material/purchases x 365 so raw material = raw material days/365 x
purchases. In this case, raw material in inventory = 3/12 x $450000 = $112500. Of course there will be
other elements such as WIP and finished goods in reality, you will need to calculate the total current
assets then minus current liabilities to get working capital.
Permanent and fluctuating current assets
1. Permanent current assets are the amount required to meet long-term minimum needs and maintain
normal trading activity. For example, inventory and accounts receivable.
18

2. Fluctuating current assets vary due to the unpredictability of business activity.


Working capital policy
Working capital policies can cover the level of investment in current assets, the way in which current
assets are financed, and the procedures to follow in managing elements of working capital such as
inventory, trade receivables, cash and trade payables. The twin objectives of working capital
management are liquidity and profitability, and working capital policies support the achievement of
these objectives.
Working capital financing
Working capital can be financed by a mixture of short (riskier but cheaper) and long-term finance
(more expensive but less risk). It depends on the working capital policy adopted by the company which
is set based on the risk attitude:
1. Conservative policy holding high levels of working capital. This approach to financing working
capital will be that all non-current assets and permanent current assets, as well as part of fluctuating
current assets are financed by long-term finance. This is safer but less profitable due to higher interest
rates.
2. Aggressive policy holding low levels of working capital to reduce finance cost and increase
profitability. This approach to financing working capital will be that fluctuating and some of the
permanent current assets are financed by short-term finance. This increases liquidity risk but cheaper.
3. Moderate policy This is a middle way between the aggressive and conservative policy, balancing
between risk and return, and follow matching principle. Long-term finance is matched with noncurrent assets and permanent current assets.
Other factors to consider
1. The industry in which the organisation operates. This is particularly important for the management
of receivables as it will be difficult to offer a shorter credit than competitors.
2. Management attitudes to risk will determine the working capital policy.
3. Previous funding decisions. If previous way was proven to be very suitable for the organisation, then
manager may follow it.
4. Organisation size. The larger size organisation will probably require more working capital.
5. Terms of trade. The terms of trade must be comparable with those of competitors and the level of
receivables will be determined by the credit period offered and the average credit period taken by
customers.

19

Investment appraisal
1. The nature of investment decisions and the appraisal process
Firstly, the distinction must be made between capital and revenue expenditure. Capital expenditure is
expenditure which results in the acquisition of non-current assets or an improvement in their earning
capacity. Revenue expenditure is incurred for the purpose of the trade of the business or to maintain
the existing earning capacity of non-current assets.
Secondly, we should differentiate between non-current assets and working capital investment.
Investment in non-current assets involved large amount of money and it takes some time for the
benefit to cover the investment cost. Non-current assets will be used on a continuing basis within the
organisation.
Investment in working capital involved a smaller amount of money and arises from the need to pay out
money for resources such as raw materials before it can be recovered from sales of the finished
product or service. It is therefore needed to maintain the operational trade cycle.
Capital budget
Capital budget is essentially a non-current asset purchase budget, and it will form part of longer term
plan of a business. Regular and minor non-current asset purchases may be covered by an annual
allowance provided for in the capital budget. Major projects will need to be considered individually and
will need to be fully appraised. Therefore, investment appraisal holds an important role in capital
budgeting.
Stages of capital investment decision-making process
1. Identifying investment opportunities Investment opportunities or proposals could arise from
analysis of strategic choices, analysis of the business environment, research and development, or legal
requirements. The key requirement is that investment proposals should support the achievement of
organisational objectives.
2. Screening investment proposals In the real world, capital markets are imperfect, so it is usual for
companies to be restricted in the amount of finance available for capital investment. Companies
therefore need to choose between competing investment proposals and select those with the best
strategic fit and the most appropriate use of economic resources.
3. Analysing and evaluating investment proposals Candidate investment proposals need to be
analysed in depth and evaluated to determine which offer the most attractive opportunities to achieve
organisational objectives, for example to increase shareholder wealth. This is the stage where
investment appraisal plays a key role, indicating for example which investment proposals has the
highest net present value.
4. Approving investment proposals The most suitable investment proposals are passed to the
relevant level of authority for consideration and approval. Very large proposals may require approval
by the board of directors, while smaller proposals may be approved at divisional level, and so on. Once
approval has been given, implementation can begin.
20

5. Implementing, monitoring and reviewing investments The time required to implement the
investment proposal or project will depend on its size and complexity, and is likely to be several
months. Following implementation, the investment project must be monitored to ensure that the
expected results are being achieved and the performance is as expected. The whole of the investment
decision-making process should also be reviewed in order to facilitate organisational learning and to
improve future investment decisions.
2. Non-discounted cash flow techniques
Relevant cash flows
The skill of identifying relevant cash flows is essential in investment appraisal, we only take into
account the relevant cash flows. Relevant cash flows are always future, incremental cash flows.
Variable cost is normally relevant and fixed cost (or unavoidable) is normally irrelevant (incremental
fixed cost is relevant). Opportunity cost (contribution loss from not taking another option) is relevant
but sometime it may be difficult to identify. The extra or reduced tax is also relevant. Interest will not
be relevant here because it is allowed in discount rate (discount rate discussed later). We will now look
at non-discounted cash flow techniques to investment appraisal, payback method and return on capital
employed (ROCE) are covered.
Payback method
The payback period is the time taken for the initial investment to be recovered by cash inflows (time
for cash inflows = cash outflows). When there are two projects, project with the least payback period is
favoured.
Example: An investment would costs $10000 and generate cash inflows of $3000 per annum, what is
the payback period?
Solution:
Year
cash flows ($)
accumulated cash flows ($)
0
($10000)
($10000)
1
$3000
($7000)
2
$3000
($4000)
3
$3000
($1000)
4
$3000
$2000
Payback period = 3 years + 1000/3000 x 12 months = 3 years 4 months.
Advantages of payback period are:
(i) It is easy to calculate and understand
(ii) Widely used in practice as a first screening method (fast check).
(iii) Identify quick cash generating projects.
Disadvantages of payback period are:
(i) Total profitability is ignored.
(ii) Time value of money is ignored.
(iii) Does not take into account positive cash inflows occurring after the end of the payback period.
21

Return on capital employed (ROCE) or accounting rate of return (ARR) or ROI


ROCE calculates estimated average annual profits/estimated average investment x 100% to evaluate
an investment. If it exceeds target rate of return, the project will be accepted. When there are two
acceptable projects, the one with highest ROCE will be favoured.
Average profit can be calculated as total profits for x year/x year (eg. Total profits for 5 years/ 5).
Average investment can be calculated as (initial investment costs + residual value)/2.
Advantages of ROCE are:
(i) A widely understood and used method, it is in percentage as well.
(ii) Use readily available accounting data.
(iii) Look at the entire project life.
Disadvantages of ROCE are:
(i) Based on accounting profits (accrual concept) rather than cash flows, it included costs like
depreciation, therefore may not be relevant to the project performance.
(ii) Does not take into account the timing of cash inflows and outflows.
(iii) Ignore time value of money.
3. Discounted cash flows (DCF) techniques, ie. net present value (NPV), internal rate of return (IRR)
Before looking at NPV and IRR, the concept of compounding, discounting and time value of money
should be understood.
Compounding
Compounding is a method of converting present value to future value by using the formula:
F = P (1 + r) ^ n, F is future value with interest, P is amount invested now (present value), r is rate of
interest in decimal, n is number of years.
Example: The cost of investment is $2000 now at 10%, what would the investment be worth after 5
years?
Solution: F = $2000 (1 + 0.10) ^ 5 = $3222.
Discounting
Discounting is a method of converting future cash flows into present value (the value now), therefore
the formula is the reverse of compounding, ie. F/[(1 + r)^n] or F x (1 + r)^(-n) but you can just use the
present value table in exam. The cost of capital (required rate of return by investors) can be used as
the discount factor.
Example: The companys cost of capital is 10% and a cash flow is expected to occur at year 2, what is
the present value rate to be used to convert this cash flow to present value?
Solution: Referring to the present value table, it is 0.826, this can be calculated:
(1 + 0.1)^(-2) = 0.826.

22

If the annual cash flows are constant from year to year, this is called annuities. When this occurs, a
short-cut will be to use annuity factors to convert the future cash flows to present values.
Example: The companys cost of capital is 10% and a constant annual cash flows of $5000 is expected
to occur from year 2 to year 5, what is the present value of the cash flows?
Solution: Referring to the annuity table, at annuity factor of 10% period 5, the present value rate is
3.791, this includes period 1s rate, so present value rate for year 2 to year 5 = present value rate of
year 5 present value rate of year 1 = 3.791 0.909 = 2.882. This can also be calculated by adding the
present value rate of year 2 to year 5 from the present value table.
Present value = $5000 x 2.882 = $14410.
In the case of perpetuity, this means that annual cash flows are constant and occur in infinite time. The
present value rate is calculated as 1/r.
Example: The companys cost of capital is 10% and cash inflows are expected to be $20000 per annum
in perpetuity, what is the present value of the cash flows?
Solution: Present value rate = 1/0.1 = 10. Present value = $20000 x 10 = $200000.
Time value of money
This is an important consideration in decision-making. Most people would prefer $100 today rather
than $100 in 10 years' time. Because $100 will probably buy you less in 10 years' time than it will today.
NPV and IRR recognise this and therefore discount the future cash flows to present value in project
appraisal.
Net present value
NPV method calculates the present value of all cash flows, and sums them to give the NPV. If this is
positive, then the project is acceptable. When performing NPV calculations, the following approach
should be taken:
1. Identify the relevant cash inflows and outflows of the project, not forgetting the initial investment.
The timing of cash flows is very important. In NPV all cash flows are assumed to occur at year ends only:
(i) If cash flows occur at the beginning of investment project, then include in time 0.
(ii) If cash flows occur during the time period, then include in this time.
(iii) If cash flows occur at the beginning of time period, then include in previous time.
2. Add up the cash inflows and outflows for each year, then discount each of the cash flows to its
present value, using the company's cost of capital.
3. Calculate the net present value of the project by adding all present values for each year.
4. Decide whether or not the project should be accepted (accept if positive NPV).
Advantages of NPV are:
(i) Positive NPV project will maximise shareholder wealth.
(ii) Takes into account the time value of money.
(iii) Based on cash flows which are less subjective than profit.
23

Disadvantages of NPV are:


(i) Can be difficult to identify an appropriate discount rate (this depends on cost of capital).
(ii) Cash flows are assumed to occur at year ends only.
(iii) Some managers are unfamiliar with the concept of NPV.
Internal rate of return
IRR tells us the rate at which the NPV of a project is zero. There are four steps to an IRR calculation:
1. Calculate the project's NPV at cost of capital (required rate of return).
2. If the above NPV is positive, choose a higher discount rate and calculate the NPV again. If the above
NPV was negative, choose a lower discount rate. This is because you need a positive and a negative
NPV to get accurate IRR (dont worry if you get two positive or negative NPV).
3. You must now calculate the IRR by using the following formula:
IRR = A + [(a/a b) x (B A)]
Where A is the lower discount rate and B is the higher rate, a is the NPV at the lower rate and b is the
NPV at the higher rate.
4. The IRR must then be compared to the company's cost of capital (required rate of return). If IRR is
higher than the required rate of return, the project should be accepted. If it is lower than the required
rate of return, the project should be rejected.
Example: Companys cost of capital is 10% and considering a project. NPV using 10% rate of return is
$1000, after calculating NPV at rate of return of 15%, NPV = ($3000). Calculate IRR.
Solution: IRR = 10 + [(1000/(1000 + 3000) x (15 10)] = 11.25%, it is higher than cost of capital of
company and therefore the project is acceptable.
Advantages of IRR are:
(i) Take into account the time value of money.
(ii) Results are expressed as simple percentage, easier to understand.
(iii) Indicates how sensitive calculations are to changes in interest rates.
Disadvantages of IRR are:
(i) May be confused with ROCE.
(ii) Problems occur if there are mutually exclusive projects (discussed below).
(iii) Some managers are not familiar with IRR method.
Other issues about NPV and IRR
1. Sometimes there are mutually exclusive projects where for example NPV is positive but IRR is lower
than cost of capital. In this case, we will take NPV as priority and accept the project.
2. An investment project may have multiple internal rates of return if it has unconventional cash
flows (ie. the pattern of cash flows that are not like starting with initial cash outflow followed by a
series of cash inflows). One solution is to use the NPV instead of IRR, since the non-conventional cash
flows are easily accommodated by NPV.

24

3. Both method uses reinvestment assumption (an assumption about the rate of return of the project

when reinvested). NPV assumes that net cash inflows generated during the life of the project can be
reinvested in the same project to earn rate of return equals to cost of capital (discount rate). IRR
assumes that net cash inflows can be reinvested in the same project to earn rate of return equals to
IRR.
Discounted payback
The approach will be similar to payback method, we are still calculating the time where investment
cost will be covered by cash inflows, but this time the cash flows will be discounted to present values.
Advantages of discounted payback method
1. Take into account time value of money.
2. Easy to calculate and understand.
3. Identify quick cash generating projects.
4. Payback can be adjusted for risk by discounting future cash flows with a risk-adjusted discount rate.
Disadvantages of discounted payback method
1. Require cost of capital to be estimated first.
2. Ignore cash flows beyond discounted payback period.
3. No decision criteria to indicate whether the investment increases companys value, unlike NPV and
IRR, eg. positive NPV means increase value, IRR more than cost of capital means acceptable.
Superiority of discounted cash flow (DCF) methods over non-DCF methods
It is the obvious point that DCF methods take into account time value of money which makes the
assessment of project more accurate. They also take into account all cash flows of the project, unlike
payback method which only looks at when investment cost is paid back.
Timing of cash flows is taken into account as opposed to ROCE method. In addition, they use relevant
cash flows in investment appraisal, ROCE used profit which includes non-cash items.
Finally, there are universally accepted methods of calculating NPV and IRR, unlike ROCE which people
can have different formula and therefore different percentage leading to different decision. Both NPV
and IRR are widely used in practice, payback and ROCE were mostly used for initial project screening
only (quick look through of how fast to pay back or profitability).
4. Allowing for inflation and taxation in DCF
Relationship between interest rates and inflation
This can seen from Fisher formula which is given in exam, (1 + i) = (1 + r)(1 + h), i = nominal or money
rate of interest, r = real rate of interest, h = inflation rate. Real rate of interest is the interest that is
adjusted for inflation while nominal rate of interest covers inflation as well. In this formula, we can see
that if inflation rises, the nominal rate of interest will also rise and this can be taken as the discount
factor to use.

25

The rule to decide whether to use real rate or nominal rate in NPV computation is as follow:
1. If cash flows are expressed in actual number of dollars that will be received or paid on the various
future dates, use nominal rate for discounting.
2. If cash flows are expressed in value of dollar at time 0 (ie. current price level), use real rate for
discounting.
Example: Inflation rate is 10% a year and company requires a minimum return of 20% on the project.
The cash flows of the project are as follow, calculate NPV.
Time
Cash flows ($)
0
(15000)
1
9000
2
8000
3
7000
Solution: Using fisher formula:
(1 + 0.2) = (1 + r)(1 + 0.1)
1 + r = 1.2/1.1, r = 9.1%
In this question, we are given cash flows are various future dates, so use nominal rate.
Time
Cash flows ($)
Discount factor (20%)
Present value ($)
0
(15000)
1
(15000)
1
9000
0.833
7497
2
8000
0.694
5552
3
7000
0.579
4053
NPV
2102
We can also use real rate to discount but we have to convert the future cash flows to value at time 0
first by discounting the future cash flows using the inflation rate as discount factor (as inflation is
related to general price level).
Time
Cash flows ($)
Discount factor (10%)
Cash flows at time 0 price
0
(15000)
1
(15000)
1
9000
0.909
8181
2
8000
0.826
6608
3
7000
0.751
5257
With this information available we can now discount the cash flows using real rate.
Time
Cash flows ($)
Discount factor (9.1%)
Present value ($)
0
(15000)
1
(15000)
1
8181
(1.091)^(-1)
7502
2
6608
(1.091)^(-2)
5553
3
5257
(1.091)^(-3)
4047
NPV
2102
You will get the same or nearly same (due to rounding difference) answer, but in exam you should
know which rate is most suitable to use in the given cash flows.

26

If the inflation rate of some cash flows is different than the general level of inflation, you just need to
inflate those cash flows specifically for each year and finally discount at nominal rate. Note that you
should not inflate those cash flows which are affected by general level of inflation as the nominal rate
has taken into account about it.
Taxation effects of relevant cash flows
In investment appraisal, tax is often assumed to be payable one year in arrears (this year tax is paid
next year). You have learnt in taxation that capital allowance (tax-allowable depreciation/writing down
allowance) can reduce the amount of tax payable, the tax benefit of the capital allowance is a relevant
cash flow (not the tax-allowable depreciation!). This tax savings can be calculated by capital allowance
x tax rate. You also know that when you sell an asset, there is either balancing charge (sale price > tax
written down value, this will be taxable profit) or balancing allowance (tax written down value > sale
price, this will be tax allowable loss), again the tax effect (eg. taxable profit x tax rate) on the sale of
asset is a relevant cash flow. In the year of disposal, capital allowance cannot be claimed. If you have
some cost savings, this will subject to extra tax payable.
Example: Psychic Co is considering whether or not to purchase an item of machinery costing $40000
payable immediately. It would have a life of 4 years, after which it would be sold for $5000. The
machinery would create annual cost savings of $14000.
The company pays tax one year in arrears at an annual rate of 30% and can claim capital allowance on
a 25% reducing balance basis. A balancing allowance is claimed in the final year of operation. The
companys cost of capital is 8%. Should the machinery be purchased?
Solution: You must be careful about the taxation effects, company pays tax one year in arrears, so for
example year 1 tax is paid at year 2 and therefore tax benefit of capital allowance is earned at year 2.
Also note that the annual cost savings will increase tax payable and for example, cost savings in year 1
mean that extra tax is paid in year 2.
Capital allowance
Tax benefits
Year
$
Year
$
1
40000 x 0.25
10000
2
10000 x 0.3
3000
2
(40000 10000) x 0.25 7500
3
7500 x 0.3
2250
3
(30000 7500) x 0.25 5625
4
5625 x 0.3
1688
23125
In year 4 (disposal), the reducing balance is $16875 (40000 23125) and the machinery is sold for only
$5000, which mean there is a tax allowable loss of $11875 (16875 5000). The tax benefit of this is in
year 5, ie. $11875 x 0.3 = $3563.
The extra tax payable for annual cost savings is 14000 x 0.3 = $4200.
Time
0
1
2
3
4
5
$
$
$
$
$
$
Machine costs
(40000)
5000
Cost savings
14000
14000
14000
14000
Extra tax on cost savings
(4200)
(4200)
(4200)
(4200)
27

Tax benefit on capital allowance


3000
2250
After-tax cash flows
(40000)
14000
12800
12050
Discount factor @ 8%
1.000
0.926
0.857
0.794
Present values
(40000)
12964
10970
9568
The NPV is $5187 so the purchase appears to be worthwhile.

1688
16488
0.735
12119

3563
(637)
0.681
(434)

When taxation is ignored in DCF calculations, the discount factor will reflect the pre-tax rate of return
required on capital investments. When tax is included then post-tax required rate of return should be
used. For example, if tax rate is 20% and the pre-tax rate of return is 10%, the post-tax rate of return is
0.1 x 0.8 = 0.08, ie. 8%.
Working capital
This is relatively simple. Increases in working capital would mean more investment in working capital,
so there will be a cash outflow. Remember, only the changes in working capital investment are cash
flows. Working capital is assumed to be recovered at the end of the project.
Example: A project lasts for 5 years with a $20000 working capital requirement at the end of year 1,
rising to $30000 at the end of year 2. State the effect of this.
Solution: $20000 cash outflow will be shown in year 1 and $10000 (30000 20000) cash outflow in
year 2. $30000 cash flow will be shown in year 5 (end of project).
5. Adjusting for risk and uncertainty in investment appraisal
There is a difference between risk and uncertainty. Risk is refers to the variability of returns and
probabilities can be estimated for the possible outcomes. Uncertainty is where probabilities cannot be
estimated objectively and will increase as the project life increases. There are a number of techniques
to deal with risk and uncertainty.
Main techniques of adjusting for risk and uncertainty in investment appraisal
An overview of the techniques is shown here, the application of sensitivity analysis and probability
analysis are shown later (examinable on calculations).
1. Sensitivity analysis this assesses the sensitivity of project NPV to changes in project variables. It
calculates the relative change in a project variable (eg. sales volume) required to make the NPV zero, or
the relative change in NPV for a fixed change in a project variable. Only one variable is considered at a
time. When the sensitivities for each variable have been calculated, the key or critical variables can be
identified. These show where assumptions may need to be checked and where managers could focus
their attention in order to increase the likelihood that the project will deliver its calculated benefits.
However, since sensitivity analysis does not incorporate probabilities, it cannot be described as a way
of incorporating risk into investment appraisal, although it is often described as such.
2. Probability analysis this approach involves assigning probabilities to each outcome of an
investment project, or assigning probabilities to different values of project variables. The range of net
present values that can result from an investment project is then calculated, together with the joint
28

probability of each outcome. The net present values and their joint probabilities can be used to
calculate the mean or average NPV (the expected NPV or ENPV) which would arise if the investment
project could be repeated a large number of times. Other useful information that could be provided by
the probability analysis includes the worst outcome and its probability, the probability of a negative
NPV, the best outcome and its probability, and the most likely outcome. Managers could then make a
decision on the investment that took account more explicitly of its risk profile.
3. Simulation this will overcome the problems of having large number of possible outcomes. A
simulation model could be constructed by assigning random numbers to each uncertain variable and
the random numbers must match their probabilities. Random numbers would be generated by
computer program and these would be used to assign values to each uncertain variable.
4. Risk-adjusted discount rate the greater the risk, the greater the risk premium required and
therefore, the discount rate should be higher. In theory, capital asset pricing model (CAPM) will be
useful in this case (CAPM discussed in cost of capital area).
5. Adjusted payback - Payback can be adjusted for uncertainty by shortening the payback period. The
logic here is that as uncertainty increases with the life of the investment project, shortening the
payback period for a project that is relatively risky will require it to pay back sooner, putting the focus
on cash flows that are more certain (less risky) because they are nearer in time. Payback can also be
adjusted for risk by discounting future cash flows with a risk-adjusted discount rate, i.e. by using the
discounted payback method. The normal payback period target can be applied to the discounted cash
flows, which will have decreased in value due to discounting, so that the overall effect is similar to
reducing the payback period with undiscounted cash flows.
Sensitivity analysis
Sensitivity of each variable = NPV/present value of project variable x 100%. The lower the percentage,
the more sensitive is NPV to that project variable, this means that small changes in that project
variable will influence NPV significantly (it might be useful to think sensitivity as margin of safety).
Management should pay attention to this project variable.
Example: Fire Co has a cost of capital of 8% and is considering a project with the following most likely
cash flows. Measure the sensitivity (%) of the project to changes in the levels of expected costs and
savings.
Year
Purchase of plant
Running costs
Savings
$
$
$
0
(7000)
1
2000
6000
2
2500
7000
Solution: Make sure you calculate the present values of each variable.
Year
Discount factor (8%) PV of plant cost PV of RC PV of S PV of net cash flow
$
$
$
$
0
1.000
(7000)
(7000)
1
0.926
(1852)
5556
3704
29

0.857

(2143)
(3995)

(7000)
Plant cost sensitivity = 560/7000 x 100% = 8%
Running cost sensitivity = 560/3995 x 100% = 14%
Savings sensitivity = 560/11555 x 100% = 4.8%. (Most sensitive)

5999
11555

3856
560

If you are required to calculate the sensitivity of cost of capital, you have to estimate one IRR (it is
better not to use cost of capital as discount factor) and then compare IRR and cost of capital, the
differences in percentage is the sensitivity.
Example: IRR has been estimated to be 18.52% and cost of capital is 8%. Measure the sensitivity of cost
of capital.
Solution: Cost of capital sensitivity = (18.52 8)/8 x 100% = 132%. This means that cost of capital can
increase by 132% before NPV becomes negative.
Also one more thing to note is that when calculating sensitivity of sales volume, NPV should be divided
by present value of contribution because changes in sales volume will affect sales and also variable
costs, ultimately affecting contribution.
Probability analysis
You have learnt that expected value = possible outcome x probability. However in F9 level, joint
probabilities might need to be used when calculating year 2 expected cash flows as year 1 cash flows
are affected by probabilities as well. Joint probabilities are calculated by multiplying both probabilities.
The following example is an extract from June 2010 question 1 with modification.
Example: Water Co has prepared the following forecasts of net cash flows for the next two periods,
together with their associated probabilities.
Period 1 cash flow
Probability
Period 2 cash flow
Probability
$000
$000
8000
10%
7000
30%
4000
60%
3000
50%
(2000)
30%
(9000)
20%
Water Co expects to be overdrawn at the start of the period 1 by $500000. Calculate:
(i) Expected value of the period 1 closing balance.
(ii) Expected value of the period 2 closing balance.
(iii) Probability of negative cash balance at the end of period 2.
(iv) Probability of exceeding the overdraft limit at the end of period 2.
Solution:
(i) It is good that you show the closing balance for each of the different cash flows because this will be
used in (ii) to add with period 2 cash flows.

30

Opening balance
$000
(500)
(500)
(500)

Cash flow
$000
8000
4000
(2000)

Closing balance
$000
7500
3500
(2500)

Probability
0.1
0.6
0.3

Expected value
$000
750
2100
(750)
2100

Expected value of period 1 closing balance = $2100000.


(ii) Be careful here, for example when the opening balance of period 2 is $7500000, there are three
possible cash flows for it. In this case, joint probabilities must be used to calculate the expected value
of period 2 closing balance.
Opening balance Cash flow Closing balance
Joint probability
Expected value
$000
$000
$000
$000
7500
7000
14500
0.1 x 0.3 = 0.03
435
7500
3000
10500
0.1 x 0.5 = 0.05
525
7500
(9000)
(1500)
0.1 x 0.2 = 0.02
(30)
3500
7000
10500
0.6 x 0.3 = 0.18
1890
3500
3000
6500
0.6 x 0.5 = 0.3
1950
3500
(9000)
(5500)
0.6 x 0.2 = 0.12
(660)
(2500)
7000
4500
0.3 x 0.3 = 0.09
405
(2500)
3000
500
0.3 x 0.5 = 0.15
75
(2500)
(9000)
(11500)
0.3 x 0.2 = 0.06
(690)
3900
The expected value of period 2 closing balance is $3900000.
(iii) With the information in (ii), you can solve (iii) and (iv) without any problem. The probability of
negative cash balance at the end of period 2 is 0.02 + 0.12 + 0.06 = 20%, ie. Just take those
probabilities that result in negative expected value.
(iv) Probability of exceeding the overdraft limits (ie. -$500000) = 0.12 + 0.06 = 18%.
6. Specific investment decisions (Lease or buy; asset replacement; capital rationing)
Leasing
Leasing is a commonly used source of finance. You should know that lessee can use the asset but the
ownership of the asset belongs to lessor. Therefore, lessor can claim capital allowance and lessee can
claim tax deduction for lease payment. There are three types of leasing:
1. Operating lease lessor is responsible for maintaining the asset and it is for shorter term.
2. Finance lease lessee is responsible for maintaining the asset.
3. Sale and leaseback lessee sells the asset and later lease back.
However in F9 we are not concerned with this, this is just for your knowledge. Also, we are normally in
the position of lessee who wants to make decision whether to lease or buy.

31

Lease or buy decisions


Lease or buy an asset is actually a financing decision as the decision whether to have the asset is an
investment decision. When we lease the asset, we can claim tax deduction on lease payment and if we
buy, we can claim capital allowance as we own the asset. To keep things simple, we should compare
the cash flows of purchasing and leasing to evaluate decision. The cash flows are discounted at an
after-tax cost of borrowing as tax is included in the cash flows. Remember the cash flows of purchasing
do not include the interest payments on the loan (if any) as cost of capital has dealt with it. However,
do not forget that there are other issues such as liquidity, flexibility, alternative use of funds etc to
consider before making the decision.
Example: Thunder Co has decided to install a new milling machine (investment decision is made). The
machine costs $20000 and it would have a useful life of five years with a trade-in value of $4000 at the
end of the 5th year. The company has two choices:
1. Purchase the machine for cash, using bank loan facilities, current rate of interest is 13% before tax.
2. Lease the machine under an agreement which would entail payment of $4800 at the end of each
year for the next five years.
The rate of tax is 30%. Capital allowance (CA) is given at 100% in year 1 if machine is purchased. Tax is
payable with a years delay. Decide whether to lease or buy the machine.
Solution: We will use after-tax cost of borrowing to discount, it is calculated as 13% x 70% = 9%. Now
we should compare the NPV of leasing and borrowing to purchase.
Borrow to purchase:
Year Item
Cash flow Discount factor (9%) Present value
$
$
0
Machine cost
(20000)
1.000
(20000)
5
Trade-in value
4000
0.650
2600
2
Tax savings on CA (20000 x 30%)
6000
0.842
5052
6
Balancing charge (4000 x 30%)
(1200)
0.596
(715)
NPV of purchase
(13063)
Note: Interest payment should not be included as it has been taken into account in the discount factor.
In year 5, the machine is traded for $4000 cash inflow, therefore creating a balancing charge of $4000,
there is an extra tax payment which is paid in year 6.
Lease
Year Item
Cash flow Discount factor (9%) Present value
$
$
1-5
Lease payment
(4800)
3.890
(18672)
2-6
Tax savings (4800 x 30%)
1440
3.569
5139
NPV of leasing (13533)
Note: There will be tax deduction given for lease payment and it is saved in the next year in this case.
You should use annuity factor to save time, to get 3.569, use year 6 present value rate at annuity factor
of 9%, ie. 4.486 minus year 1 present value rate, ie. 0.917.
By comparing the NPV, purchasing the machine will be the cheaper option.
32

The above example is for the lessee, if you are a lessor, lease payment will be your income which is
also taxable. You will buy the asset to lease to the lessee so the cost of asset becomes your cash
outflow but you might be able to claim capital allowance since you are the owner of the asset.
Therefore, if you are the lessor, the evaluation of whether to lease or not depends on the NPV of
leasing to the lessee.
Asset replacement decisions
The decision is to decide how frequently an asset should be replaced with an identical asset. The
equivalent annual cost method can be used to calculate an optimum replacement cycle. Equivalent
annual cost is the average cost of owning an asset over its entire life. We have to first calculate the
present value of cost over one replacement cycle and then convert it to equivalent annual cost by
dividing a suitable present value rate, ie. PV of cost/annuity factor (annuity table is useful).
Example: Megatron Co operates a machine purchased at $25000 which has the following costs and
resale values over its three year life.

Year 1
Year 2
Year 3
$
$
$
Running costs (cash expenses)
7500
11000
12500
Resale value (end of year)
15000
10000
7500
The organisations cost of capital is 10%. Assess how frequently the asset should be replaced.
Solution:
Replace every year Replace every 2 years Replace every 3 years
Year Discount factors Cash flow
PV
Cash flow
PV
Cash flow
PV
$
$
$
$
$
$
0
1.000
(25000) (25000)
(25000) (25000)
(25000) (25000)
1
0.909
(7500)
(6818)
(7500) (6818)
(7500)
(6818)
15000
13635
2
0.826
(11000) (9086)
(11000) (9086)
10000
8260
3
0.751
(12500) (9388)
7500
5633
PV of cost over one
replacement cycle
(18183)
(32644)
(44659)
Replacement every year:
Equivalent annual cost = (18183)/0.909 = $(20003)
Replacement every 2 years:
Equivalent annual cost = (32644)/1.736 = $(18804), note 1.736 = 0.909 + 0.826 or you can get this from
annuity table.
Replacement every 3 years:
Equivalent annual cost = (44659)/2.487 = $(17957)
33

From these we can see that the equivalent annual cost of replacement every 3 years is the least and
this would mean that optimum replacement policy is to replace the machine every 3 years.
Single period capital rationing
Capital rationing is where a company has a limited amount of money to invest and investments have to
be compared in order to allocate monies most effectively. Capital rationing may occur due to internal
factors (soft capital rationing, eg. reluctant to issue additional share capital, reluctant to raise
additional debt capital, wish only to use retained earnings, reserve capital for projects to be taken in
another period, capital may be needed for other necessary expenditures such as replacement of noncurrent assets) or external factors (hard capital rationing, eg. depressed stock market, restrictions on
bank lending, issue costs, financial position does not attract investors).
Profitability index (PI)
When capital rationing occurs in a single period, projects are ranked in terms of PI to determine the
mix of projects. PI can only be applied to divisible projects (possible to undertake a fraction of a
project), if the projects are non-divisible (discussed later) then do not apply PI. PI = NPV/present value
of capital investment. Therefore, this compares the NPV per investment cost of each project and those
with high PI will be ranked first. However you should exclude the investment cost from NPV, generally
PI of 1.0 is the lowest acceptable index.
Example: There are three projects, A, B and C with investment cost of $20000, $30000 and $40000
respectively and present value of cash inflows of $30000, $35000 and $41000 respectively. The
company has only $55000 available for investment. The projects are divisible. Evaluate how the
projects are to be undertaken.
Solution: Firstly we calculate the PI for each of the projects.
PI for A = 30000/20000 = 1.5
PI for B = 35000/30000 = 1.17
PI for C = 41000/40000 = 1.025
The ranking is first given to A, then B and finally C. After investing for A and B, company still has $5000
to invest for C, which is 12.5% of the investment cost. The decision is therefore to invest fully in A and
B and 12.5% of C.
However note that this method ignores the size of the projects, strategic value and the cash flow
patterns.
Trial and error
For non-divisible projects (impossible to undertake a fraction of a project, eg. building ship), trial and
error would have to be used to test the NPV available from different combinations of projects. For
example, when there are three projects, A, B and C, we will have to calculate the NPV of A + B, A + C
and B + C, the combination with the highest NPV will be undertaken.

34

Business finance
1. Sources of and raising short-term finance
The common sources of short-term finance include overdraft, short-term loan, trade credit and lease
finance.
Overdraft
Overdrafts are subject to an agreed limit and must be paid if bank demand for repayment (repayable
on demand). Overdraft is commonly used as a support for normal working capital, eg. to increase the
current assets or to reduce other current liabilities. The customers only pay interest when they are
overdrawn (credit balance of bank account). The advantage of this is that it wont affect financial
gearing of the company and is cheap.
Short-term loan
It is drawn in full at the beginning of the loan period and repaid at a specified time or in installments.
Loan is actually more suitable for medium term purpose. Interest must be paid unlike overdraft. Once
the loan is agreed, the term of the loan must be adhered to. The advantage of this is that it helps in
better planning since the amount of interest and repayments is known. But note that it will affect the
gearing calculation.
Trade credit
This is one of the main sources of short-term finance as it is like an interest-free borrowing. However,
there will be a risk of losing supplier goodwill and also cost of not accepting early settlement discounts.
Lease finance
Leasing is a good source of finance to acquire an asset through finance lease (long-term finance) or
renting an asset for a while through operating lease (short-term finance). Company can also sell an
asset to a lessor and lease back (sale and leaseback) to get immediate cash (short or long-term finance).
Types of leases are covered earlier.
2. Sources of and raising, long-term finance
The common sources of long-term finance include equity finance, debt finance and venture capital.
Equity finance
This is raised through the issue of ordinary shares to investors. The ordinary shareholders will then able
to attend company general meetings and vote. This means that they have some control of the business.
Debt finance
In simplest meaning, debt financing means borrowing money on credit with a promise to repay the
amount borrowed, plus interest. It is usually in the form of bonds or debentures. It is useful to
differentiate equity and debt finance first before looking at types of debt finance:
35

(i) Cost the cost of equity is higher than the cost of debt. This is because an equity investor takes a
greater risk. If the company goes into liquidation, an equity investor is the last person to be paid any
money. Therefore, an equity investor expects a higher return to reflect the risk he is taking. Debt
finance is cheaper as interest payments are tax deductible but dividends (for equity finance) are not.
Debt finance also has lower risk.
(ii) Control of the business equity is normally invested into the business through the issue of ordinary
shares. Shareholders will share the ownership of the business and carry voting rights. Hence, a
shareholder can participate in business decisions. Debt finance avoids the share of control (company
will still have full control).
(iii) A significant difference between debt and equity is that debt has to be repaid, whereas equity does
not.
(v) Effect on gearing The more debt finance is raised, the higher is the gearing level. The higher equity
finance is raised, the lesser is the gearing level. Gearing = debt/equity or debt/(equity + debt).
Now lets consider different types of debt finance.
Deep discount bonds
These are issued at a price which is at a large discount to the nominal/face value of the loan notes.
They will be redeemable (repaid) at par (or above par) when they eventually mature. This would mean
that investors can get a large capital gain (redemption value issue price) when redeemed. However,
the interest rate is much lower than other types of bonds.
Zero coupon bonds
These are similar to deep discount bonds. They are issued at large discount to their face value and no
interest is payable. Investors gain from the difference between redemption value on maturity and the
issue price. These are also known as zero interest bonds.
Convertible bonds
These give the holder the right to convert to other securities (normally ordinary shares) at a predetermined price and time. The smarter investors will determine whether the conversion rights are
attractive or not by estimating a conversion premium. Conversion premium = current market value
current conversion value.
Example: Leaf Co quoted 10% convertible bonds at $142 per $100 nominal (this means it has a nominal
value of $100 which we normally assumed). The earliest date for conversion is in four years time, at
the rate of 30 ordinary shares per $100 nominal bond. The share price is currently $4.15. Decide
whether the conversion rights are attractive or not.
Solution: We should first calculate the conversion value which is 30 x $4.15 = $124.50. This is the value
of the converted shares if conversion is done.
Conversion premium = $142 - $124.50 = $17.50.
This would mean that if the conversion rights have to be attractive, the share price would have to rise
by $17.50 or 14% (17.50/124.50).
36

We can also estimate the market value of convertible bond by discounting the future cash flows of it.
Example: Rock Co quoted 10% convertible bonds with nominal value of $100, interest is payable yearly.
Each $100 of convertible bonds may be converted into 40 ordinary shares in three years time or
redeemed at $110 per $100 nominal value of bond. Assume the required pre-tax rate of return is 8%
and investors will redeem at maturity, estimate the likely current market price for $100 of the bonds.
Solution:
Cash flow
Discount factor (8%) Present value
Year
$
$
$
1
Interest (10% x 100)
10
0.926
9.26
2
Interest
10
0.857
8.57
3
Interest
10
0.794
7.94
Redemption value
110
0.794
87.34
Estimated market value of $100 debt
113.11
Venture capital
Venture capital is risk capital, normally provided in return for an equity stake. Venture capitalists are
prepared to invest in new businesses, small businesses, specific expansion projects or management
buyouts (managers purchase all or parts of a business). They will be less interested in providing the
money required to finance running expenditure and working capital requirements. Also, venture
capitalists will want to involve in running the business because of their need to protect their
investment. Venture capitalists will take into account certain factors in deciding whether or not to
invest:
(i) The nature of companys product the selling potential of products.
(ii) Expertise in production technical ability to produce efficiently.
(iii) Expertise in management commitment, skills and experience.
(iv) Market and competition threat from current competitors and also future new competitors.
(v) Future profits they will want to see the detailed business plan.
(vi) Board membership they will ensure that they are part of representatives of the board of directors
and have say in future strategy.
(vii) Exit routes they will consider potential exit routes in order to realise the investment.
Methods of raising equity finance
To be able to issue shares to the public, company has to go for stock market listing which allows
company to access to wider pool of finance and improve marketability of shares. The unquoted
company can obtain a listing on stock market through initial public offer (IPO) or placing.
Initial public offer
When companies go public for the first time, a large issue will probably takes the form of an IPO. This
is known as flotation. Subsequent issues are likely to be placing or rights issue (discussed later).
However the costs of share issue include underwriting cost, stock market listing fee, professional fees
and advertising cost.
37

Underwriting cost is the cost incurred to get an underwriter who will subscribe those shares which are
not taken-up by the public. Therefore, underwriter is a risk taker.
Placing
Placing means arranging for most of an issue to be brought by a small number of institutional investors
and this is cheaper than IPO.
Setting a share issue price
This will depend on a number of factors such as price of similar quoted companies, current market
conditions, future trading prospects and whether there is a desire for immediate premium. Company
should avoid over-pricing (to avoid undersubscribed) or under-pricing an issue (avoid oversubscribed).
Rights issues
Rights issue is an offer to existing shareholders enabling them to buy more shares, usually at a price
lower than current market price. With this, company does not have to issue to public which is more
costly and can dilute voting rights of existing shareholders. Rights issue can affect EPS as it affects the
number of ordinary shares.
Example: Shock Cos profit after tax is always 20% of capital employed. Directors propose to raise an
additional $126000 from a rights issue. Current market price is $1.80 so it is decided that the rights
price is $1.60. Before the rights issue, the capital employed is $300000 (with 200000 ordinary shares of
$1 each). Calculate the dilution in EPS with the rights issue.
Solution: Earnings before rights issue = 20% x $300000 = $60000. EPS before rights issue =
60000/200000 x 100 = 30c.
Capital employed after rights issue = $426000. Therefore, earnings after rights issue = 20% x 426000 =
$85200.
Number of new shares from rights issue = $126000/$1.60 = 78750.
EPS after rights issue = $85200/278750 x 100 = 30.6c
Dilution of EPS = 30.6 30 = 0.6 cents.
For the shareholders to take up the rights issue, they may consider the value of the rights. Theoretical
ex rights price (TERP, market price after the rights issue in theory) is an important consideration. Value
of rights can be determined by TERP issue price.
Example: Thunder Co decides to make a rights issue of one new share at $1.50 each for every four
shares already held. After announcement of the use, the share price fell to $1.95, but by the time just
prior to the issue being made, it had recovered to $2 per share (actual cum rights price). Calculate TERP
and discuss whether shareholder would exercise the rights.
Solution: As TERP is in theory, the calculation will seem theoretical.

38

4 shares @ $2
$8.00
1 share @ $1.50
$1.50
5
$9.50
It is estimated that 5 shares are worth $9.50, so TERP = 9.5/5 = $1.90. The value of rights = $1.90 $1.50 = $0.40. This means that shareholder can expect to gain $0.40 for each new share he buys so he
might exercise the rights.
The actual ex rights price (actual market price after rights issue) may differ from TERP. The market will
take a view of how profitable the new funds will be invested and will value the shares accordingly.
1. If new funds are expected to generate earnings at the same rate as existing funds, actual ex rights
price will probably be the same as TERP.
2. If new funds are expected to generate earnings at a lower rate, actual ex rights price < TERP.
3. If new funds are expected to generate earnings at higher rate than current funds, actual ex rights
price should rise above the TERP.
Shareholders actions with regard to rights issues
1. Do nothing % of ownership will reduce as others subscribe the shares.
2. Sell the rights - % of ownership will reduce.
3. Fully subscribe - % of ownership will be maintained.
4. Any combination of the above actions.
Example: King, an owner of 1000 shares in Queen Co, has been offered rights issue of 1 for 5 with issue
price of $3.40 and current market price of companys shares is $4 per share. Theoretical ex-rights price
(TERP) is determined as $3.90. Value of rights is $0.50 ($3.90 - $3.40). Identify the actions that King
could take and the effect of these actions on the wealth of King.
Solution: Number of shares from rights issue = 1000/5 = 200.
Value of 1,200 shares after rights issue = 1,200 x 390 = $4680
Value of 1,000 shares before rights issue = 1,000 x 400 = $4000
Value of 1,000 shares after rights issue = 1,000 x 390 = $3900
Cash subscribed for new shares = 200 x 340 = $680
Cash raised from sale of rights = 200 x 0.50 = $100
1. Do nothing
It is obvious that King will lose $100 (4000 3900) in wealth if he ignores the rights issue.
2. Sell the rights
In this case, the wealth of King = $3900 + $100 = $4000. Therefore, part of the wealth has changed
from shares to cash, but there is neutral effect on Kings wealth (original is $4000).
3. Fully subscribe
By subscribing in full, Kings wealth becomes $4680 (4000 + 680). This is the same as the value of 1200
shares after rights issue, so there is neutral effect on Kings wealth.
39

3. Raising short and long-term finance through Islamic financing


Islamic finance is based on Islamic law, ie. Shariah. The main principles include:
1. Wealth must be generated from legitimate trade and asset-based investment. (The use of money for
the purposes of making money is expressly forbidden.)
2. Investment should also have a social and an ethical benefit to wider society beyond pure return.
3. Risk should be shared.
4. All harmful activities (haram) should be avoided. (eg. investing in drug business)
Concept of interest (riba)
Charging and receiving interest is prohibited in Islamic finance because it represents the money itself
being used to make money (Not in accordance to first principle above). Therefore, when Islamic banks
provide finance they must earn their profits by other means. This can be through a profit-share relating
to the assets in which the finance is invested, or can be via a fee earned by the bank for services
provided. The essential feature of Shariah is that when commercial loans are made, the lender must
share in the risk. If this is not so then any amount received over the principal of the loan will be
regarded as interest.
Islamic financial instruments
1. Murabaha is a form of trade credit/loan for asset acquisition that avoids the payment of interest.
Instead, the bank buys the item and then sells it on to the customer on a deferred basis at a price that
includes an agreed mark-up for profit. The mark-up is fixed in advance and cannot be increased, even if
the client does not take the goods within the time agreed in the contract. Payment can be made by
instalments. The bank is thus exposed to business risk because if its customer does not take the goods,
no increase in the mark-up is allowed and the goods, belonging to the bank, might fall in value.
2. Ijara is a lease finance agreement whereby the bank buys an item for a customer and then leases it
over a specific period at an agreed amount. Ownership of the asset remains with the lessor (bank)
regardless of operating or finance transaction, which will seek to recover the capital cost of the
equipment plus a profit margin out of the rentals payable.
3. Mudaraba is essentially like equity finance in which the bank and the customer share any profits.
The bank will provide the capital, and the borrower, using their expertise and knowledge, will invest
the capital. Profits will be shared according to the finance agreement (no dividends are paid), but as
with equity finance there is no certainty that there will ever be any profits, nor is there certainty that
the capital will ever be recovered. This exposes the bank to considerable investment risk as losses are
solely attributable to the provider of capital. In practice, most Islamic banks use this is as a form of
investment product on the liability side of their statement of financial position, whereby the investor
or customer (as provider of capital) deposits funds with the bank, and it is the bank that acts as an
investment manager (managing the funds).
4. Sukuk (Islamic bonds) is debt finance which cannot bear interest. Typically, an issuer of the Sukuk
would acquire property and the property will generally be leased to tenants to generate income. The
sukuk are issued by the issuer to the Sukuk holders, who thereby acquire a proprietary interest in the
assets of the issuer. The issuer collects the income and distributes it to the Sukuk holders. This entitles
40

to a share of the income generated by the assets. Most Sukuk give Sukuk holders ownership of the
cash flows but not of the assets themselves.
5. Musharaka is a joint venture or investment partnership between two parties. Both parties provide
capital towards the financing of projects and both parties share the profits in agreed proportions. This
allows both parties to be rewarded for their supply of capital and managerial skills. Losses would
normally be shared on the basis of the equity originally contributed to the venture. Because both
parties are closely involved with the ongoing project management, banks do not often use Musharaka
transactions as they prefer to be more hands off.
4. Internal sources of finance and dividend policy
Internal sources of finance
Internal sources of finance include retained earnings and increasing working capital management
efficiency:
1. Retained earnings company can choose to retain the earnings or paid out as dividends. Retained
earnings belong to shareholders and are classed as equity financing. Retained earnings are a flexible
source of finance, enabling directors to invest without asking for approval by financial providers. There
is no new share issue so no issue cost and no dilution of control. However, shareholders may be
sensitive to the loss of dividends and there is an opportunity cost of retaining the earnings (cash can be
invested to earn more rather than keeping them idle). Retained earnings are no doubt, the most
important single source of finance for companies.
2. Increasing working capital management efficiency there will be savings that can be generated from
more efficient management of trade receivables, inventory, cash and trade payables.
Relationship between dividend policy and financing decision
When deciding how much dividends to be paid out to shareholders, one of the main considerations is
the amount of earnings to retain to meet financing needs. A company must restrict its self-financing
through retained earnings because shareholders should be paid a reasonable dividend.
Factors influencing dividend policy
Legal constraints
The law on distributable profits could require company to pay dividends solely out of accumulated net
realised profits or restrict the amount of dividends payable.
Profitability
A company cannot consistently pay dividends higher than its profit after tax as a healthy level of
retained earnings is needed to finance the continuing business needs of the company.
Liquidity
The company must have enough cash to pay the dividends it declares and the desire to hold cash
would mean lower dividend payout.

41

Shareholder expectations
Shareholders usually expect a consistent dividend policy from the company, with stable dividends each
year or a steady dividend growth. Stable dividends or steady dividend growth are usually needed for
share price stability.
Signaling effect
Dividend declared can be interpreted as a signal from directors to shareholders about the strength of
underlying project cash flows or the future prospects of the company. For example, a cut in dividends
may be treated by shareholders as signaling that the future prospects of the company are weak.
Therefore, companies tend to adopt a stable dividend policy.
Theories of dividend policy
Residual theory
Company should invest any project with positive NPV and dividends should be paid only when these
investment opportunities are exhausted.
Traditional view
Shareholders prefer large dividends to smaller dividends because the dividend is sure but future capital
gains are uncertain.
Irrelevancy theory by Modigliani and Miller (MM)
Shareholders are not concerned with the dividend policy since they can sell a portion of their portfolio
of equities if they want cash. This indicates that an issue of dividend should have little or no impact on
share price, perfect capital market is assumed. MM proposed that in a tax-free world, shareholders are
indifferent between dividends and capital gains, and the value of a company is determined solely by
the earning power of its assets and investments.
Other basic terms
1. Scrip dividends dividend is paid in the form of new shares rather than by cash.
2. Scrip issue/bonus issue issue of new free shares to existing shareholders, by converting equity
reserves to share capital.
3. Stock split for example, each ordinary share of $1 each is split into two shares of 50c each. This
creates cheaper shares with greater marketability.
4. Share repurchase company could purchase its own shares when there is surplus cash or to increase
EPS (reduce number of shares). This could also prevent a takeover or enable a quoted company to
withdraw from the stock market.
5. Gearing and capital structure considerations
Problem of high levels of gearing
Gearing is the amount of debt finance a company uses relative to its equity finance. The greater the
level of debt, the more is the financial risk. Financial risk can be seen from different points of view:
42

1. Company when the debts are too high that company cannot pay back, it will be forced into
liquidation. Therefore, gearing can measure the companys ability to remain in business.
2. Lenders lenders will want higher interest yield to compensate the higher financial risk faced.
3. Ordinary shareholders similar to lenders. They would want a higher expected return from their
shares to compensate for higher financial risk faced. Market value of shares will therefore depend on
gearing.
In short, gearing increases variability of shareholder earnings and risk of financial failure.
Ratio analysis
Ratios can be used to assess the impact of sources of finance on financial position and financial risk.
The relevant ratios include:
1. Financial gearing = long-term debt/capital employed x 100% or market value of long-term
debt/(market value of equity + market value of long-term debt) x 100%. This will indicate the financial
risk. Long-term debt/equity is also a method to calculate financial gearing.
2. Operational gearing = contribution/profit before interest and tax (PBIT). This will indicate the
business risk. For example, if contribution is high but PBIT is low, fixed costs will be high so business
risk will be high. Other ways to calculate include fixed cost/total cost or fixed cost/variable cost.
3. Interest coverage ratio = PBIT/interest. Generally, an interest coverage ratio of less than three times
is considered low, indicating that profitability is too low given the gearing of the company.
4. Debt ratio = total long-term debts/total assets. This indicates the financial risk as well.
Note for financial and business risk
1. Business risk/operating risk possibility of changes in level of profit before interest as a result of
changes in turnover or operating costs. Business risk relates to the nature of business operations.
2. Financial risk/gearing risk possibility of changes in level of distributable earnings as a result of the
need to make interest payments on debt finance or prior charge capital.
Cash flow forecasting
A change in sources of finance can be taken into account in cash flow forecast. If the change in sources
of finance results in significant cash deficit, then company might need to consider other, more
appropriate source of finance.
Effect of gearing on shareholder wealth
A company will only be able to raise finance if shareholders think the returns they can expect are
satisfactory in view of the risks they are taking. If a company can generate returns on capital in excess
of the interest payable on debt, financial gearing will raise the EPS and this affects two ratios:
1. Price earnings ratio (P/E ratio) = market price per share/EPS. This reflects the markets appraisal of
the shares future prospects.
2. Dividend cover = EPS/dividend per share.
Another important ratio is dividend yield which is dividend per share/market price per share x 100%.
With this, we look at the effect of gearing on market price of shares. The changes in dividend yield
43

required by shareholders (required rate of return) can impact the market price of the shares (discussed
more in business valuation topic).
6. Finance for small and medium sized entities (SMEs)
SMEs can be defined as having three characteristics:
1. Firms are likely to be unquoted.
2. Owned by s small number of individuals, typically a family group.
3. They are not micro-businesses (very small businesses that exist to employ just owner).
They are always in need of finance to run or expand the business. However, they have some problems
in obtaining finance.
Financing problems of SMEs
SMEs may not know about the sources of finance available or difficult to obtain finance because of the
risks faced. More specific problems are:
1. Funding gap there is a high failure rate so hard to raise external finance and there are few
shareholders so hard to raise internal finance.
2. Maturity gap it is difficult for SMEs to obtain medium-term loans due to mismatching of the
maturity of assets and liabilities. Longer-term loans are easier to obtain than medium-term loans as
longer loans are easily secured with mortgages against property.
3. Inadequate security banks will be unwilling to increase loan funding without an increase in security
given (which the owners may be unwilling or unable to give).
Government aids to ease financing problems of SMEs
UK government aid includes:
1. Loan guarantee scheme help businesses to get a loan from the bank because bank would be
unwilling to lend as SMEs cannot offer the security that the bank would want.
2. Grants sum of money given to an individual or business for a specific project or purpose. A grant
usually covers only part of the total costs involved.
3. Enterprise capital funds (ECFs) designed to be commercial funds, investing a combination of
private and public money in small high-growth businesses. Each ECF will be able to make equity
investments of up to 2m into eligible SMEs.
Appropriate sources of finance for SMEs
1. Equity owners personal resources or those of family connections are generally the initial sources
of finance. Since the business will have few tangible assets at this stage, it will be difficult to obtain
equity from elsewhere. The external investors would not like to invest because of the inability of SMEs
to offer an exit route.
2. Overdraft financing interest may be more expensive as bank takes the high risk.
3. Bank loans likely to be available only for projects or assets which are in long-term.
4. Trade credit taking extended credit from suppliers is a source of finance for many SMEs. However
this might cause loss of early settlement discounts and loss of supplier goodwill.
44

5. Business angel financing business angels are wealthy individuals who invest directly in small
businesses. However the amount of money available from individual angels may be limited. Business
angels generally have prior knowledge of the industry. As business angel financing occurs in informal
market, it is difficult to arrange with the business angels.
6. Leasing.
7. Factoring.
8. Venture capital venture capitalists are prepared to invest in new businesses and specific expansion
projects. However, venture capitalists will want to involve in running the business because of their
need to protect their investment.

45

Cost of capital
1. Sources of finance and their relative costs
Now, you should understand that every sources of finance have costs, because by raising capital using
these sources, company is required to return an amount to satisfy the providers of funds. Therefore,
cost of capital can be interpreted as the required rate of return (%) by the providers of funds.
Remember that cost of capital can be used in investment appraisal.
Cost of equity and debt
We can split cost of capital into cost of equity and cost of debt. Cost of debt is likely to be lower than
the cost of equity because debt is less risky. Lets look at the creditor hierarchy, the first one will mean
least risky and the last one is the most risky one:
1. Creditors with a fixed charge (secured by specific asset such as land).
2. Creditors with a floating charge (secured by fluctuating class of assets such as receivables account).
3. Unsecured creditors.
4. Preference shareholders (being paid fixed dividend and get paid first in the event of liquidation
compared to ordinary shareholders).
5. Ordinary shareholders.
The cheapest type of finance is debt and the most expensive type of finance is equity. This determines
the differences in cost of equity and cost of debt.
2. Estimating the cost of equity
There are two ways to estimate cost of equity, dividend valuation/growth model and capital asset
pricing model (CAPM).
Dividend valuation/growth model
Dividend valuation model will be used if future dividend per share is expected to be constant, the cost
of equity is therefore the dividend yield, ie. Dividend per share/market price per share.
However, dividend normally increases year by year, so dividend growth model is needed. With this the
dividend yield formula takes into account the growth rate of the dividend, ie. cost of equity = [dividend
per share x (1 + g)/market price per share] + g, g is the growth rate.
Example: A share has a current market value of 96c, and the last dividend was 12c. If the expected
annual growth rate of dividends is 4%, calculate the cost of equity.
Solution: Cost of equity = [12 x (1 + 0.04)/96] + 0.04 = 0.17 = 17%.
Estimating growth rate
There are two ways, either through analysis of the growth in dividends over the past few years (trend)
or Gordons growth model.
By looking at the trend, we can say that dividend in year 1 x (1 + g)^4 = dividend in year 5, ie. 4 years
growth.
46

Example: Dividend in 20X1 is $150000 and dividend in year 5 is $262350, calculate the growth rate.
Solution: $150000 x (1 + g)^4 = $262350
(1 + g)^4 = 1.749
1 + g = 1.15
Growth rate = 15%. Every year dividend will increase by 15%.
Gordons growth model gives a formula g = br where b is the proportion of profits retained and r is the
rate of return on new investments. This formula is given in exam.
Example: Leaf Co retains 65% of its earnings for capital investment projects it has identified and these
projects are expected to have an average return of 8%. The current dividend per share is 15c and
market value per share is $1, calculate the cost of equity and use Gordons growth model is estimate
the growth rate in dividend.
Solution: g = 65% x 8 = 5.2%.
Cost of equity = (0.15 x 1.052/1) + 0.052 = 0.2098 = 21%.
Weaknesses of dividend growth model
1. Does not consider risk.
2. The growth rate is only an approximation, dividend do not grow smoothly in reality.
3. Assume there are no issue costs for new shares.
Capital asset pricing model
This model incorporates risk. It assumes that all investors hold diversified portfolios and as a result only
seek compensation (return) for the systematic/market risk (risk that cannot be diversified away) of an
investment. Beta factor measures a shares volatility and systematic risk is measured through this. If a
share price were to risk or fall at double the market rate, it would have a beta factor of 2.0.
The individual components of CAPM are risk-free rate of return, equity/market risk premium and
equity beta. CAPM is the cost of equity as the sum of risk-free rate of return and a risk premium
reflecting the systematic risk of an individual company relative to the systematic risk of the stock
market. This risk premium is the product of the companys equity beta and equity risk premium.
CAPM formula is therefore created as
=
where
is cost of equity,
is
the risk-free rate of return, is the equity beta of the individual security (eg. shares),
is the rate
of return on a market portfolio. Equity risk premium is therefore represented by
and the
risk premium is represented by
. CAPM formula is given in exam.
Example: Shares in Fire Co has a beta of 0.9. The expected returns to the market are 10% and the riskfree rate of return is 4%. What is the cost of equity?
Solution: Cost of equity = 4 + 0.9 (10 4) = 9.4%.
Remember that total risk is divided into systematic and unsystematic risk, beta only measures the
systematic risk.
47

Disadvantages of CAPM
1. Unrealistic assumptions.
2. Beta can change over time.
3. Need to determine the equity premium (historical returns are often used in practice).
4. Need to determine the risk-free rate (risk-free investment might be a government security).
Summary of CAPM
Since investors will expect the risk they took to be compensated with more return, CAPM takes this
into account by adding an amount of risk premium to the risk-free rate of return to increase the cost of
equity. It is usually suggested that the CAPM offers a better estimate of the cost of equity than the
dividend growth model.
3. Estimating cost of debt and other capital instruments
When company uses debt to finance capital, there will be cost of debt. We will focus on calculating
cost of debt for irredeemable debt, redeemable debt, convertible debt and preference shares, cost of
bank debt is just the current interest rate. Since interest is tax deductible, we will always take after-tax
cost of debt. Note that we normally assume the nominal value of bond is $100.
Irredeemable debt
Since the debt is irredeemable, interest is paid in perpetuity, interest yield represents the cost of
irredeemable debt, ie. Interest/market price of bond.
Example: Water Co issued bonds of $100 nominal value with annual interest of 9% based on the
nominal value. The current market price of the bonds is $90. What is the cost of the bonds?
Solution: Cost of debt = 9/90 = 10%.
If interest is not paid annually, there would be a need to express the interest yield to annual
percentage.
Example: Rock Co has 12% irredeemable bonds in issue with a nominal value of $100. The market price
is $95 ex interest. Calculate the cost of debt if interest is paid half-yearly.
Solution: 6% is payable half-yearly, so 6/95 is the cost of debt for 6 months, to express it in annual cost:
Cost of debt = (1 + 6/95)^(12/6) 1 = 13%.
Remember that interest can be allowable for tax purpose, so when there is tax (which is normally the
case), the cost of debt should be after-tax, calculated by updating interest yield formula as [interest x
(1 tax rate)]/market price of bond.
Example: Using the first example, when the tax rate is 30%, calculate cost of the bonds.
Solution: Cost of debt = (9 x 0.7)/90 = 7%.

48

Redeemable debt
The debt will be redeemed in the year of redemption and interest (allowable for tax) plus capital (not
allowable for tax, this is the nominal value of the bond) will be paid to the debt holder. In this case,
internal rate of return (IRR) of the redeemable debt represents the cost of debt. Remember that IRR is
the rate at which NPV of the cash flows will be zero.
Example: Fire Co has outstanding $660000 of 8% bonds on which the interest is payable annually on 31
December. The debt is due for redemption at par on 1 January 20X6. The market price of the bonds at
28 December 20X2 was $95. Effective tax rate was 30% and tax is paid each 31 December. Calculate
the after-tax cost of debt.
Solution: Firstly, lets try 5% as the discount factor first.
Year
Item
Cash flows
Discount factor (5%) Present value
0
Market value
(95)
1.000
(95)
1-3
Interest (8 x 0.7) (until 31/12/X5) 5.6
2.723
15.2
3
Redemption
100
0.864
86.4
NPV
6.6
Since this NPV is positive, another rate to choose must be higher in order to result in negative NPV.
Lets take 10% as the discount factor.
Year
Item
Cash flows
Discount factor (10%) Present value
0
Market value
(95)
1.000
(95)
1-3
Interest (8 x 0.7) (until 31/12/X5) 5.6
2.487
14
3
Redemption
100
0.751
75.1
NPV
(5.9)
After-tax cost of debt = 5 + [(6.6/(6.6 + 5.9) x (10 5)] = 7.6%.
The market value of the bond can also be estimated by calculating the NPV of the cash flows
discounted at cost of debt.
Example: Using the above example, if cost of debt would rise to 12% during 20X3 and 20X4, estimate
the market value of the bonds at 28 December 20X2.
Solution: It is probable that the market value in December 20X2 will reflect the new rates in future, so
we can take 12% as the discount factor.
Year
Item
Cash flows
Discount factor (12%) Present value
0
Interest (31/12/X2)
5.6
1.000
5.6
1-3
Interest (until 31/12/X5)
5.6
2.402
13.5
3
Redemption
100
0.712
71.2
NPV
90.3
The estimated market value of the bonds would be $90.3.
From the above two examples, the calculation is based on the view of the debt holder, who lends to
the company at first (so market value of the bond is included in year 0) and later receive interest and
finally capital repayment at the year of redemption.
49

Convertible debt
The cost of debt calculation will depend on whether or not conversion is likely to happen.
1. If conversion is not expected, the bond is treated as redeemable debt so use IRR to calculate cost of
debt.
2. If conversion is expected, IRR will still be used, but follow the number of years to conversion and the
redemption value is replaced by the conversion value (market value of the shares converted from
debt). Conversion value will also take growth in share price into account and can be calculated as
current share price x (1 + growth rate)^n x shares received on conversion, n is the number of years to
conversion.
Example: Fire Co issued 8% convertible bonds which are due to be redeemed in five years time. They
are currently quoted at $82 per $100 nominal. The bonds can be converted into 25 shares in five years
time. The share price is currently $3.50 and is expected to grow at a rate of 3% per annum. Assume tax
rate is 30%, calculate the cost of debt.
Conversion value = future share price x number of shares received = [$3.50 x (1.03)^5] x 25 = $101.44.
Since redemption value is $100, it is likely that investors will convert as conversion value is higher.
Year
Item
Cash flows
Discount factor (8%) Present value
0
Market value
(82)
1.000
(82)
1-5
Interest (8 x 0.7)
5.6
3.993
22.36
5
Conversion value
101.44
0.681
69.08
NPV
9.44
A higher rate will be chosen to discount, lets take 12%.
Year
Item
Cash flows
Discount factor (12%) Present value
0
Market value
(82)
1.000
(82)
1-5
Interest (8 x 0.7)
5.6
3.605
20.19
5
Conversion value
101.44
0.567
57.52
NPV
(4.29)
Cost of debt = 8 + [(9.44/(9.44 + 4.29) x (12 8)] = 10.75%.
Preference shares
For irredeemable preference shares, future cash flows are the dividend payments in perpetuity.
Therefore, we can use dividend yield to calculate the cost of debt, ie. dividend/market price of
preference share capital. Remember that dividend does not attract tax relief.
For redeemable preference shares, cost of debt is calculated using IRR.
4. Estimating the overall cost of capital
Weighted average cost of capital (WACC)
WACC is the average cost of companys finance weighted by the total market value or book value of
each source of finance. There are two methods to weight the source of finance, ie. market value and
book value weightings. Market values should always be used unless unavailable (unquoted company)
because book value is based on historical costs.
50

The formula for WACC is given in exam as

, V is the market value, T is

tax rate, e is equity, d is debt, k is cost. Since the formula includes (1 T) which means after-tax, the
cost of debt to be included, ie.
must be before-tax cost of debt.
Example: The following information relates to Rock Cos cost of finance and corporate tax rate is
currently 30%.
Source
Cost of finance
Market value ($m)
Equity
17%
23
Preference shares
18.6%
5
Debt
8.6%
14
42
The above cost of finance is before-tax. Calculate the WACC for Rock Co.
Solution: Do not combine cost of preference shares with the cost of debt, we will modify the formula.
WACC =

9.31% + 1.55% + 2% = 12.86%.

You dont actually need to refer to the formula, lets use the above example again but with different
approach and more in line with the definition of WACC.
Example: The following information relates to Rock Cos cost of finance and corporate tax rate is
currently 30%.
Source
Cost of finance
Market value ($m)
Equity
17%
23
Preference shares
18.6%
5
Debt
8.6%
14
42
The above cost of finance is before-tax. Calculate the WACC for Rock Co.
Solution: Make the cost of finance after-tax and do the following.
Source
Cost of finance
Market value ($m)
Cost x market value
Equity
17%
23
3.91
Preference shares
13%
5
0.65
Debt
6%
14
0.84
42
5.40
WACC = 5.4/42 x 100% = 12.86%. The weighting is done at cost of finance x market value.
Marginal cost of capital
The word marginal tells you about variable. It is calculated when there are changes to the cost of
finance and market value of the source of finance. It is argued that WACC should be used to evaluate
projects. However where gearing levels fluctuate significantly or finance for new project carries a
significant different level of risks, it is good to use marginal cost of capital to evaluate new project. The
following example is an extension of the above example.
51

Example: Rock Co decided to invest in a new project which will increase the cost of equity to 18% and
cost of debt to 10%. The market value of equity will then be $24m and market value of debt will be
$20m. Calculate new WACC and marginal cost of capital.
Solution:
Source
Cost of finance
Market value ($m)
Cost x market value
Equity
18%
24
4.32
Preference shares
13%
5
0.65
Debt
10%
20
2.00
49
6.97
New WACC = 6.97/49 x 100% = 14.2%.
Marginal cost of capital = (6.97 5.4)/(49 42) x 100% = 22.4%.
WACC for investment appraisal
WACC can be used as discount rate in investment appraisal when:
1. Project has the similar business risk as the existing operations of the company.
2. Project has the similar financial risk as the existing operations of the company.
3. Project should be small in comparison with the size of the company.
Be prepared to face question where cost of equity, cost of debt and WACC come together. Theoretical
aspect must not be ignored.
5. Capital structure theories and practical considerations
Traditional view of capital structure and its assumptions
Some believe that optimal capital structure (mix of equity and debt finance) will be the point at which
WACC is lowest. Cost of equity will rise when financial risk increases. As the level of gearing increases,
cost of debt remains unchanged up to a certain level of gearing, after this level the cost of debt will
increase. The assumptions used in this theory include:
1. Company pays out all its earnings as dividends.
2. Taxation is ignored.
3. Earnings and business risk are constant in perpetuity.
4. No issue cost of issuing debt or shares.

Ko = WACC. X = Optimal capital structure.


52

Miller and Modigliani (MM) view on capital structure


MM however says that WACC is not influenced by changes in capital structure. MM made various
assumptions in arriving at this conclusion, including:
1. Debt is risk-free and freely available at the same cost to investors and companies.
2. No tax or issue costs.
3. A perfect capital market exists.
MMs no tax model (assume no tax)
No optimum capital structure exists. WACC will remain constant at all levels of gearing. Therefore,
company cannot reduce its WACC by altering its gearing. In short, we can say that benefits of cheaper
debt are counterbalanced by increased cost of equity (due to increased financial risk).

MMs with tax model


MM admitted corporate tax into their analysis and conclusion changed. As debt became even cheaper
(due to tax relief on interest payments), cost of debt falls significantly from Kd to Kd (1 T). WACC will
fall when gearing increases. Therefore, if company wishes to reduce its WACC, it should borrow as
much as possible. In short, we can say that benefits of cheaper debt now exceeds the increased cost of
equity. Optimum capital structure is 99.99% debt finance.

Market imperfections
Companies are discouraged from following MMs with tax model approach to capital structure as MM
does not take into account factors such as bankruptcy costs, agency costs and tax exhaustion.

53

Bankruptcy costs
MMs theory assumes perfect capital markets so a company can always raise finance and avoid
bankruptcy. In reality, the higher the gearing level, bankruptcy risk increases. Shareholders and debt
holders will then require a higher rate of return as compensation which increase cost of equity and
debt.
Agency costs
As gearing increases, debt holders would want to impose more constrains on the management to
safeguard their increased investment. They might impose restrictive covenants in the loan agreement
which for example set the minimum level of liquidity of the company to restrict companys freedom.
Debt holders who are concerned about the increased gearing level will incur more agency costs (cost to
monitor the agent).
Tax exhaustion
Companies become tax exhausted when interest payments are no longer tax deductible as additional
interest payments exceed profits made. In this case, cost of debt will rise from Kd (1 T) to Kd.
Pecking order theory
This theory states that firms will prefer retained earnings to any other source of finance, and then will
choose debt, and last of all equity. The order of preference will be:
1. Retained earnings.
2. Straight debt.
3. Convertible debt.
4. Preference shares.
5. Equity shares.
Company follows pecking order theory because:
1. Minimise issue costs retained earnings have no issue costs, issuing debt is cheaper than issuing
equity.
2. Minimise time and expense involved in persuading outside investors.
3. Some managers believe that debt issues have a better signaling effect than equity issues. There is an
information asymmetry where managers (insiders) know more about companies prospects than the
outsiders. Therefore, market depends on the signal shown by the company to interpret about the
company. For example, market will interpret debt issues as a sign of confidence, that businesses are
confident of making sufficient profits to fulfill their obligations on debt and that they believe that the
shares are undervalued.
6. Impact of cost of capital on investments
Company value and cost of capital
The market value of a company depends on its cost of capital. The lower a companys WACC, the
higher the NPV of its future cash flows and therefore the higher will be the company value.

54

Using WACC in investment appraisal


This is covered before, WACC is suitable when:
1. Project being appraised is small relative to the company.
2. The existing capital structure will be maintained so that project has the same financial risk as the
company.
3. The project has the same business risk as the company. Therefore, WACC can be used to evaluate an
expansion of existing business.
Using CAPM in determining project-specific cost of capital
The capital asset pricing model (CAPM) can be used to calculate a project-specific discount rate in
circumstances where the business risk of an investment project is different from the business risk of
the existing operations of the investing company.
The first step in using the CAPM to calculate a project-specific discount rate is to find a proxy
(representative) company that undertake operations whose business risk is similar to that of the
proposed investment. The equity beta of the proxy company will represent both the business risk and
the financial risk of the proxy company. The effect of the financial risk of the proxy company must be
removed (ungearing) to give a proxy beta representing the business risk alone of the proposed
investment. This beta is called an asset beta.
The asset beta representing the business risk of a proposed investment must be adjusted to reflect the
financial risk of the investing company, a process called regearing. This process produces an equity
beta that can be placed in the CAPM in order to calculate a required rate of return (a cost of equity).
This can be used as the project-specific discount rate for the proposed investment if it is financed
entirely by equity. If debt finance forms part of the financing for the proposed investment, a projectspecific weighted average cost of capital can be calculated.
Ungearing and regearing of beta
As discussed above, ungearing beta means from a equity beta of the proxy company to an asset beta
and regearing beta means from asset beta to equity beta of the investing company. Formula for asset
beta is given in exam as
(which follow MMs view) and its beta (

. We often assume debt is risk-free


) is then taken as zero, so the formula is reduced to

, if without tax then remove the (1 T). To convert the ungeared beta (asset
beta) to geared beta (equity beta), simply modify the formula of asset beta to

Example: Two companies are identical in every respect except for their capital structure. Water Co has
a debt to equity ratio of 1:3 and its equity has a value of 1.20. Fire Co has a debt to equity ratio of 2:3.
Currently Fire Co is considering a project with risk-free rate of return of 4% and equity risk premium of
6%. Calculate the specific-project cost of equity for Fire Cos project.
Solution: We will start with ungearing the equity beta of Water Co. Note that it is not necessary to put
market value of equity or debt in $ inside the formula.
55

Next we regear this to equity beta for Fire Co.

We can now apply this to the CAPM formula to determine specific-project cost of equity.
Cost of equity =
= 4 + 1.427 x 6 = 12.6%.
Limitations of beta formula
1. Difficult to identify a proxy company with identical operating characteristics.
2. Estimates of beta values from share price information are not wholly accurate as they are based on
statistical analysis of historical data.
3. There may be differences in beta values between firms caused by different size of the organization
and debt capital not being risk-free.

56

Business valuations
1. Nature and purpose of the valuation of business and financial assets
There are a number of different ways of putting a value on a business or on shares of unquoted
company. A share valuation will be necessary when (examples):
1. In a takeover bid (another company (A) wishes to acquire this company (B)), offer price might be
estimated at fair value which is in excess of current market price of Bs shares in Stock Exchange.
2. Unquoted company wishes to go public, issue price of the shares have to be decided.
3. Holding company wishes to sell subsidiary and is negotiating the price.
4. Company is in a liquidation situation.
Information required for valuation
There are many, some are listed below:
1. Past financial statements information.
2. Aged accounts receivable summary.
3. List of marketable securities.
4. Inventory summary.
5. Budgets for a minimum of five years.
6. List of major customers.
However there are limitations to some of the information. For example, some information may be out
of date or subjective.
Market capitalisation
This is an important term representing an estimation of the value of business obtained by current price
per share x number of ordinary shares.
2. Models for the valuation of shares
Three models to look into, ie. asset-based, income-based and cash flow-based valuation models.
Asset-based valuation models
Net assets valuation method provides a lower limit for the value of a company. By itself it is unlikely to
produce the most realistic value. In this model, value of share = (total assets total liabilities)/number
of ordinary shares, ie. net assets per share. Intangible assets are excluded as they do not represent the
worth of companys physical assets, but if they have a market value (eg. license which could be sold),
then they can be included.
The choice of asset values to use includes:
1. Net book value (statement of financial position basis) unlikely to be realistic.
2. Net realisable value suitable if assets are to be sold or business as a whole broken up.
3. Net replacement cost suitable if assets are to be used on an on-going basis.

57

Use of net asset valuation


1. As a measure of the security (safety) in a share value as it provides a lower limit of share value.
2. As a measure of comparison in a scheme of merger (business combination which nonce obtains
control over any other) company can compare this value with another company in a merger to
determine whether it is necessary to adjust the value of the company to allow for difference.
3. As a floor value (minimum value) for a business that is up for sale.
Advantages of net asset valuation
1. Information is readily available.
2. Indicate a minimum value of the company.
Disadvantages of net asset valuation
1. Ignore future profitability expectations.
2. Statement of financial position valuations depend on accounting conventions, which might be very
different from market valuations.
Income-based valuation models
We can use either price-earnings ratio (P/E ratio) method (commonly asked) or earnings yield method.
It mainly involves playing with the formula.
P/E ratio method
P/E ratio = market value of share/earnings per share, therefore the market value per share can be
determined as P/E ratio x EPS. However it may not be suitable to use P/E ratios of quoted companies to
value unquoted companies, in exam you should normally take a figure around one half to two thirds of
the industry average when valuing an unquoted company. Factors to consider in deciding suitable P/E
ratio include:
1. General economic and financial conditions.
2. Industry eg. if P/E ratio is affected by lack of confidence in the industry, the valuation will be
unrealistically low.
3. Financial status of any principal shareholders.
4. Reliability of profit estimates and the past profit record.
5. Asset backing (net asset value) and liquidity.
Earnings yield method
Earnings yield = EPS/market value per share, therefore the market value per share = EPS/earnings yield.
We can also incorporate earnings growth, then earnings yield = ([EPS x (1 + g)]/market value per share)
+ g, and market value per share = [EPS x (1 + g)/(earnings yield g).
Cash flow-based valuation models
We can use either dividend valuation model/dividend growth model (commonly asked) or discounted
cash flow basis. It involves playing with the formula for dividend valuation/growth model.
58

Dividend valuation model/dividend growth model


It is covered before in cost of equity topic that when dividend growth model is used, cost of equity =
cost of equity = [dividend per share x (1 + g)/market price per share] + g, therefore market price per
share = [dividend per share x (1 + g)]/(cost of equity g). Cost of equity can be determined using
capital asset pricing model (CAPM). Now it is good to look at December 2007 question 1 (a).
Example: Phobis Co is considering a bid for Danoca Co. Both companies are stock-market listed and are
in the same business sector. Financial information on Danoca Co, which is shortly to pay its annual
dividend, is as follows:
Number of ordinary shares
5 million
Ordinary share price (ex div basis)
$3.30
Earnings per share
40.0c
Proposed payout ratio
60%
Dividend per share one year ago
233c
Dividend per share two years ago
220c
Equity beta
14
Other relevant financial information:
Average sector price/earnings ratio
10
Risk-free rate of return
46%
Return on the market
106%
Required:
Calculate the value of Danoca Co using the following methods:
(i) price/earnings ratio method;
(ii) dividend growth model;
and discuss the significance, to Phobis Co, of the values you have calculated, in comparison to the
current market value of Danoca Co.
Solution: (i) Market value per share = P/E ratio x EPS = 10 x 40 = 400c = $4.
Value of Danoca = $4 x 5m = $20m.
(ii) Market value per share = [dividend per share x (1 + g)]/(cost of equity g), therefore we need to
find the missing figures.
Dividend per share = 40c x 60% = 24c.
We can estimate the growth by finding out the average geometric dividend growth rate, 22c x (1 + g)^2
= 24c, (1 + g)^2 = 24/22, so g = (24/22)^1/2 1 = 4.5%.
We are given enough information to use CAPM to find cost of equity.
Cost of equity = 4.6 + 1.4 x (10.6 4.6) = 13%.
Market value per share = [24 x (1 + 0.045)]/(0.13 0.045) = $2.95.
Value of Danoca = $2.95 x 5m = $14.75m.
59

Current market capitalisation of Danoca Co is $16.5m ($3.3 x 5m). The P/E ratio value of Danoca Co is
$20m, higher than the current market capitalisation due to the difference in P/E ratio of the average
sector and Danoca Co. This indicates that there is a need to improve financial performance of Danoca
Co after acquisition so that Phobis Co and its shareholders will benefit from the acquisition.
Dividend growth model value is less than current market capitalisation of Danoca Co. Dividend growth
model value represents the minimum value that Danoca shareholders will accept if Phobis Co makes an
offer to buy their shares. This difference may reflect the belief of stock market that a takeover bid is
imminent as shareholders are prepared to accept a value lower than market capitalisation but it could
also be due to inaccuracy of cost of equity or expected dividend growth rate. (Referred to answer)
The dividend models made a number of assumptions:
1. Investors act rationally and homogeneously failed to take into account that expectations of
shareholders may be different.
2. Other influences on share prices are ignored.
3. Companys earnings will increase sufficiently to maintain dividend growth levels.
4. Dividends either show no growth or constant growth.
5. No increase in cost of capital.
One obvious problem is when valuing company that doesnt pay dividends.
Discounted cash flow basis
Appropriate when one company intends to buy the assets of another company and to make further
investments in order to improve cash flows in the future. Value of investment = expected after-tax
cash flows discounted to present values at appropriate cost of capital. This value of investment will be
the maximum price that company is willing to pay for the shares of another company.
3. The valuation of debt and other financial assets
Again we will play with the formulas of the cost of debt here. We always assume that debt is quoted
with $100 nominal value.
Irredeemable debt
The cost of irredeemable debt, as mentioned before, is the same as interest yield. When there is tax,
cost of debt = [interest x (1 T)]/market price of bond ex interest, so market price of bond = [interest x
(1 T)]/cost of debt. Remember that ex interest means excluding any interest payment that might
soon be due.
Redeemable debt
Valuation of redeemable debt depends on future expected receipts (interest and redemption).
Therefore, the market price of bond = NPV of the cash flows.

60

Example: Fire Co issued some 9% bonds, which are now redeemable at par in three years time.
Investors now require a redemption yield of 10%. What will be the current market value of each $100
of bond?
Solution:
Year
Items
Cash flows ($) Discount factor (10%) Present value ($)
1-3
Interest
9
2.487
22.38
3
Redemption
100
0.751
75.10
NPV
97.48
Each $100 of bond has a current market value of $97.48.
Convertible debt
If conversion is not expected, then market value of bond = NPV of cash flows. If conversion is expected,
then market value of bond is still the NPV of cash flows, but follow the number of years to conversion
and also redemption value is replaced with conversion value (refer back to cost of debt topic if you are
confused).
Example: Water Co issued 9% convertible bond which can be converted in five years time into 35
ordinary shares or redeemed at par on the same date. An investors required return is 10% and the
current market price of the underlying share is $2.50 which is expected to grow by 4% per annum.
Estimate the current market value of convertible bond assuming that conversion will occur.
Solution: Conversion value = 35 x $2.5 x (1.04)^5 = $106.46.
Year
Items
Cash flows ($) Discount factor (10%) Present value ($)
1-5
Interest (9% x $100)
9
3.791
34.12
5
Conversion
106.46
0.621
66.11
Current market price of convertible bond
100.23
Preference shares
Remember that tax is not allowed for the fixed dividend payment. Cost of preference shares = dividend
payment/market price of bond so market price of bond = dividend/cost of preference shares.
4. Efficient Market Hypothesis (EMH) and practical considerations in the valuation of shares
Efficient Market Hypothesis (EMH)
EMH is the hypothesis that the stock market reacts immediately to all the information that is available.
Stock market efficiency usually refers to the way in which the prices of traded financial securities
reflect relevant information. Types of efficiency include:
1. Operational efficiency requires that transaction costs are low and do not hinder investors in the sale
or purchase of securities.
2. Informational efficiency means that relevant information is widely available at low cost.
3. Pricing efficiency refers to the ability of capital markets to process information quickly and
accurately, and arises as a consequence of operational efficiency and informational efficiency.

61

4. Allocative efficiency means that capital markets are able to allocate available funds to their most
productive use and arises as a result of pricing efficiency.
Most of the research into market efficiency has been into pricing efficiency.
There are three forms of efficiency: weak, semi-strong and strong-form efficiency.
Weak-form efficiency
This is when research indicates that share prices fully and fairly reflect past information. Since new
information arrives unexpectedly, share price movements should follow a random walk. Investors
cannot predict share prices by analysing past information because of the random walking share price.
Semi-strong form efficiency
This is when research indicates that share prices fully and fairly reflect public information (published)
as well as past information. Investors cannot generate abnormal returns by analysing either public
information, such as published company reports, or past information, since research shows that share
prices respond quickly and accurately to new information as it becomes publicly available. Therefore,
individuals cannot beat the market by reading the newspapers or annual reports, since the
information contained in these will be reflected in the share price. Evidence suggests that most leading
stock markets are semi-strong efficient.
Strong-form efficiency
This is when research indicates that share prices fully and fairly reflect all information, which includes
private information. Even investors with access to insider information cannot generate abnormal
returns in such a market. Stock markets are not held to be strong form efficient. It is unlikely that
strong-form exists in reality.
Consequences of market efficiency
1. There is no right or wrong time to issue new shares since share prices are always fair (in theory).
2. Managers will not be able to deceive the market by the timing or presentation of new information,
such as annual reports or analysts briefings, since the market processes the information quickly and
accurately to produce fair prices. Managers should therefore simply concentrate on making financial
decisions which increase the wealth of shareholders.
3. There are no bargains to be found in capital markets (in theory).
Practical considerations in the valuation of shares and businesses
A number of factors to consider in relation to shares valuation.
Fundamental analysis theory of share values
It states that the realistic market price of a share can be derived from a valuation of estimated future
dividends. Value of share will be the present value of all future expected dividends discounted at the
shareholders cost of capital.

62

Marketability and liquidity of shares


In financial markets, liquidity is the ease of buying and selling the shares without significantly moving
the price. Large companies have better liquidity and greater marketability than small companies. In
such case, the small companies may try to improve the marketability of their shares with stock split.
Availability and sources of information
This depends on the efficiency of the financial market. Efficient market is one where the prices of
securities bought and sold reflect all the relevant information available. Efficiency relates to how
quickly and accurately prices adjust to new information.
Market imperfections and pricing anomalies
Various types of anomaly appear to support the views that irrationality often drives the stock market,
including:
1. Seasonal effects may cause share prices to rise or fall.
2. A short-run overreaction to recent events, for example, stock market crash in 1987 (Black Monday)
causes huge loss in value in a very short time.
Market capitalisation
Market capitalisation or size of a company has also produced some pricing anomalies. The return from
investing in smaller companies is greater than average return from all companies in the long run,
probably due to greater risk associated with smaller companies.
Investor speculation and behavioural finance
Investor speculation means that the investor buys the shares when he expects the share price will go
up in future (now low) and sells shares when he expects the share price is at peak already (so
speculator is like a gambler). Speculators can affect the share price through these buying and selling
activities.
Behavioural finance is a psychology-based theory. It states that the characteristics of market
participants can influence the market outcomes, so the irrational investor behaviour may significantly
affect share price movement.

63

Risk management
1. The nature and types of risk and approaches to risk management
In F9, we deal with foreign currency risk and interest rate risk. We start with some introductions.
Types of foreign currency risk
1. Translation risk this risk occurs when company has a foreign branch or subsidiary and the currency
of foreign subsidiarys country weaken. It is the risk that when company consolidates accounts, there
will be exchange losses when the accounting results of foreign subsidiary are translated into home
currency. (IAS 21 requires holding company to translate the account of foreign subsidiary to home
currency in consolidation).
2. Transaction risk this arises when company is importing or exporting. It is the risk of adverse
exchange rate movements between the date the price is agreed and the date cash is received/paid,
arising during normal international trade. For example, in June a UK company agrees to sell an export
to Australia for 100,000 Australian $ (A$), payable in three months. The exchange rate at the date of
the contract is A$/1.80 so the company is expecting to receive 100,000/1.8 = 55,556. If, however,
the A$ weakened over the three months to become worth only A$/2.00, then the amount received
would be worth only 50,000.
3. Economic risk this refers to the effect of exchange rate movements on the international
competitiveness of a company and refers to the effect on the present value of future cash flows. For
example, if a company is exporting (lets say from the UK to a eurozone country) and the euro weakens
from say /1.1 to /1.3 (getting more euros per pound sterling implies that the euro is less valuable,
so weaker) any exports from the UK will be more expensive when priced in euros. So goods where the
UK price is 100 will cost 130 instead of 110, making those goods less competitive in the European
market.
Types of interest rate risk
1. Gap exposure this arises from the risk of adverse changes in interest rates between the time
interest is paid for debt and the time interest is earned from investments. Companys exposure to
interest rate risk can be identified using gap analysis (grouping together assets and liabilities which are
sensitive to interest rate changes according to their maturing dates). A negative gap occurs when
company has larger amount of interest-sensitive liabilities maturing than it has interest-sensitive assets
maturing at the same time while positive gap is the opposite of this. Therefore, with a negative gap,
company faces exposure if interest rates rise by the time of maturity. With positive gap, company faces
exposure if interest rates fall by maturity.
2. Basis risk this is the risk when yields (interest receivable) on assets and costs (interest payable) of
liabilities are based on different bases, such as LIBOR (London Inter-Bank Offered Rate, this is the rate
of interest applied to wholesale money market lending between London banks) versus U.S. prime rate.
Different bases may move at different rates or in different directions, affecting the amount receivable
or payable (not applicable for fixed interest rate debt, only apply to floating rate).

64

2. Causes of exchange rate differences and interest rate fluctuations


Before starting, there are some terms to introduce here:
1. Exchange rate rate at which one countrys currency can be traded in exchange for another
countrys currency. For example, A$ per 2.00 means that every A$1 can be exchanged for 2.00.
2. Spot rate rate of exchange agreed for immediate transactions.
3. Forward rate rate agreed NOW for currencies to be exchanged at a future date.
4. Forward contract agreement to exchange different currencies at a specified future date and at a
specified rate. These are traded privately.
Causes of exchange rate fluctuations
In general, exchange rate can be influenced by inflation, interest rates, balance of payments,
speculation and government policy on managing or fixing exchange rates. The following headings are
the causes of exchange rate fluctuations in detailed (can be confusing).
Balance of payments
Balance of payments are surplus if source of funds (eg. export goods sold) exceed uses of funds (eg.
paying for imported goods) and deficit if the other way round. As government is able to influence
exchange rate through exchange rate policy, when there is trade deficit, government will rectify by
trying to bring about a fall in the exchange rate. Similarly to prevent a balance of trade surplus from
getting too large, government will try to bring about a limited rise in the exchange rate. Government
will try to maintain a stable balance of payments.
Purchasing power parity (PPP) theory (relationship between exchange rates and inflation rates)
This suggests that changes in exchange rates over time must reflect relative changes in inflation
between two countries. PPP is based on the of law of one price which suggests that, in equilibrium,
identical goods should sell at the same price in different countries, and that exchange rates relate
these identical values. For example, if a basket of goods costs 100 in the UK and $150 for an
equivalent in the US, for equilibrium to exist, the exchange rate would be expected to be 1 = $1.50.
PPP holds in the longer term rather than the shorter term and so is often used to provide long-term
forecasts of exchange rate movements, for example for use in investment appraisal.
Interest rate parity (IRP) theory (relationship between exchange rates and interest rates)
This states that the difference between the nominal interest rates in two countries is the difference
between forward rate and the spot rate of their currencies. When one makes two fixed investments in
two different currencies, the return from both investments will be the same even though interest
rates may be different in absolute terms. This applies if there is no arbitrage (activity of buying
currency in one financial market and selling it at a profit in another). Both the spot rate and the
forward rate are available in the current foreign exchange market, and the forward rate can be
guaranteed by using a forward contract.

65

Fisher effect
This has been covered earlier that the relationship between inflation rate and interest rate are:
(1 + i) = (1 + r)(1 + h) where i is nominal/money interest rate, r is the real interest rate and h is the
inflation rate.
International Fisher effect
This states that differences in nominal interest rate between countries provide an unbiased predictor
of future changes in spot exchange rates. The international Fisher effect can be expressed as:
where

is the nominal interest rate in country a,

is the inflation rate in country a.

Four-way equivalence
This model states that in equilibrium, differences between forward and spot rates, differences in
interest rates, expected differences in inflation rates and expected change in sport rates are equal to
one another (dont worry about the formula first).

Revision
To refresh your mind, maybe it is a good idea to look back at the relationship between interest rate,
inflation rate and exchange rate.
1. When inflation rate increases, interest rate will be increased so that demand for money will fall (this
discourages people to spend). Inflation is generally caused by people spending more.
2. When exchange rate is low, interest rate can be increased to attract foreign investors to invest which
might help to improve the currency values and therefore the exchange rate.
Forecasting exchange rates
We will use PPP and IRP to forecast exchange rates.
Purchasing power parity
Expected spot rate is forecast from the current spot rate by multiplying the ratio of expected inflation
rates in the two countries being considered. Therefore, the formula is:
66

, this formula is given in exam. Country c is the foreign country.


Example: The spot exchange rate between UK sterling and Danish krone is 1 = 8.00 kroner. Assuming
there is now purchasing parity, an amount of a commodity costing 110 in the UK will cost 880 kroner
in Denmark. Over the next year, price inflation in Denmark is expected to be 5% while inflation in UK is
expected to be 8%. Calculate the expected spot exchange rate at the end of the year.
Solution: We take 8 kroner as spot rate, we are UK so country c will be Denmark.
Expected spot exchange rate = 8 x 1.05/1.08 = 7.78. This amount can also be found by:
UK price = 110 x 1.08 = 118.80, Denmark price = Kr880 x 1.05 = Kr924
New spot exchange rate = 924/118.80 = 7.78.
Interest rate parity
Forward rate is forecast from the current spot rate by multiplying the ratio of interest rates in the two
countries being considered. Therefore, the formula is:
, this formula is given in exam as well. Country c is the foreign country.
Example: Exchange rates between two currencies, the Northland florin (NF) and Southland dollar ($S)
are as follow:
Spot rates
$S1 = NF4.7250
NF1 = $S0.21164
90 days forward rates
NF4.7506 per $S1
$S0.21050 per NF1
Money market interest rate for 90 day deposits in Northland florins is 7.5% annualised. Estimate the
interest rates in Southland.
Solution: Since we are given the 90 days forward rates, the interest rate of 7.5% should be adjusted to
90 days.
Northland interest rate on 90 day deposit = 0.075 x 90/365 = 1.85%.
There is a need to adjust the formula so that forward rate/spot rate = ratio of interest rates. If we take
$S0.21164 as spot rate, we are Northland so country c is Southland.
0.21050/0.21164 = 1 + /1.0185, 1 + = 0.21050/0.21164 x 1.0185, 1 + = 1.013, so = 1.3% OR
4.7506/4.7250 = 1.0185/1 + , 1 + = 1.0185 x 4.7250/4.7506, 1 + = 1.013 so = 1.3%.
In annual rate, 0.013 x 365/90 = 5.3%.
Causes of interest rate fluctuations
The causes of interest rate fluctuations include the structure of interest rates and yield curves and
changing economic factors. The following headings are the causes in detailed.

67

Structure of interest rates and yield curves


It is quite common sense that higher risk borrowers must pay higher rates to compensate the risk that
lenders are taking. Also, larger deposits with the bank might attract higher rates than smaller deposits.
Furthermore, different types of financial asset attract different rates of interest.
Duration of the lending can affect the interest rates as well, we refer to the term structure of interest
rates, ie. yield on a security varies according to the term of the borrowing, this is shown by the yield
curve below:

Yield can mean interest rate/market price (running yield) or the discount rate that makes present value
of future interest payments and redemption value equal to current market price, ie. IRR (redemption
yield or cost of debt). Long-term debt is usually expected to be more expensive than short-term debt
(ie. yield curve slopes upward, meaning the lender can yield more for longer-term debt). This is
because lender faces greater risk that borrower may not be able to repay interest payments or
principal amount.
Expectations theory
Forward interest rate is due only to expectations of interest rate movements. If interest rates are
expected to fall, short-term rates might be higher than long-term rates (so that lender maintains
sufficient yield %) and yield curve would be downward sloping.
Liquidity preference theory
Liquidity preference means investors/lenders prefer cash now and want compensation in the form of
higher return for being unable to use their cash now. Therefore, the long-term interest rates do not
only reflect investors assumptions about future interest rates but also include a premium to
compensate investors for added risk.
Market segmentation theory (segmented market theory)
This theory states that most investors have set preferences regarding the length of maturities that they
will invest in (investors are interested in bonds of only one maturity). For example, bank prefers shortterm debt and pension funds prefer long-term debt. Slope of the yield curve will reflect conditions in
different segments of the market. Even if short term rate increases in any period of time, this theory
implies that investors will not shift from long term bonds to short term bonds in order to enjoy higher
rate in the short run.

68

3. Hedging techniques for foreign currency risk


Hedging is a risk management strategy to limit or offset probability of loss from fluctuations in eg.
currencies and securities. Remember, hedging is not about maximising income, it is about providing
certainty about the cost of a transaction in your own currency.
Foreign currency risk management methods (managing transaction risks)
Currency of invoice
Arrange for the contract and the invoice to be in your own currency. This will shift all exchange risk
from you onto the other party.
Netting
Netting is a process where credit balances are netted off against debit balances so that only the
reduced net amounts remain due to be paid by actual currency flows. If you owe your US supplier
US$1m, and another US company owes your US subsidiary US$1.1m, then by netting off group
currency flows your net exposure is only for US$0.1m. Bilateral netting is where two companies in the
same group cooperate as explained above; multilateral netting is where many companies in the group
liaise with the groups treasury department to achieve netting where possible.
Matching
Company can reduce or eliminate foreign exchange transaction exposure by matching receipts and
payments. Wherever possible, company that expects to make payments and have receipts in same
foreign currency should plan to offset its payments against its receipts in the same currency, opening a
foreign currency bank account will help. For example:
1 November: should receive US$2m from US customer
15 November: must pay US$1.9m to US supplier.
Deposit the US$2m in a US$ bank account and simply pay the supplier from that. That leaves only
US$0.1m of exposure to currency fluctuations.
Leading and lagging
Company can try to lead payments (payments in advance) when expect foreign currency will
strengthen against own currency and lag payments (delaying payments) when expect that foreign
currency will weaken against own currency.
Forward exchange contracts
This is a binding agreement to sell or buy an agreed amount of currency at a specified time in the
future at an agreed exchange rate (the forward rate). Each spot and forward there is always a pair of
rates given. For example:
Spot
/ 1.2025 0.03 (ie. 1.2028 and 1.2022)
Three-month forward rate
/ 1.2020 0.06 (ie. 1.2026 and 1.2014)
Bank will always give you the rate that is more favourable to them, ie. 1.2022 if changing to euros
now, or 1.2014 if using a forward contract.
69

Example: A UK importer imports from a foreign seller on 1 April for 26500 Swiss francs and has to pay
in one months time, on 1 May. He can arrange a forward exchange contract with his bank on 1 April,
whereby the bank undertakes to sell the importer 26500 Swiss francs on 1 May, let say forward rate of
SFr2.6400 = 1. With this, the UK importer must pay at this forward rate on 1 May, amount payable is
26500/2.6400 = 10037.88.
1. If spot rate on 1 May is lower than 2.6400, the importer would have successfully protected himself
against the weakening of sterling.
2. If spot rate on 1 may is higher than 2.6400, importer would have to pay more. The extra cost is
unavoidable because forward contract is binding.
Let say later the seller fails to deliver the goods as specified, the forward exchange contract must still
be satisfy. With this, bank will sell the customer 26500 Swiss Francs at 10037.88 and buy back the
unwanted currency at the spot rate. Importer could win or lose depending on the spot rate at the time.
This process is called closing out where bank arranges for customer to perform his part of the foreign
exchange contract by either selling or buying the missing currency at spot rate.
Money market hedging
Money market hedging is the use of borrowing and lending transactions in foreign currencies to lock in
the home currency value of a foreign currency transaction. This involves borrowing in one currency,
converting the money borrowed into another currency and putting the money on deposit until the
time the transaction is completed, hoping to take advantage of favourable exchange rate movements.
Example will be required to understand how it works.
1. Foreign currency receipt (aim to create a foreign currency liability and use the foreign currency
receipt to pay it).
Example: A UK manufacturer exported to a US company and in 3 months time he will receive US$2m.
To face no currency risk, this US$2m can be used to settle a US$2m liability. UK manufacturer can
create a US$2m liability by borrowing US$2m now and repaying that in 3 months time with the
US$ receipt. Interest rates information is important, let say US$ 3 months interest rate is 0.54% - 0.66%
(this amount is always annualised). Same rule, UK manufacturer will be charged at higher rate, ie. 0.66%
per annum. The amount that UK manufacturer actually need to borrow now is:
X (1 + 0.66%/4) = 2000000, X = US$1996705. This can be changed now from US$ to at the current
spot rate, say US$/ 1.4701, to give 1358210 (1996705/1.4701). This 1,358,210 is certain, UK
manufacturer can deposit it into the bank or use it elsewhere.
Finally, when US$2m is received from US company, it will be used to repay the loan which should have
increased to US$2m from US$1996705.
2. Foreign currency payment (aim to create a foreign currency asset and use this to pay the foreign
currency payment, cost involved is the borrowings to create the foreign currency asset).

70

Example: A UK company owes a Danish creditor Kr3500000 in three months time. Spot exchange rate
is Kr7.5509 Kr7.5548 per 1. The company can borrow in Sterling for 3 months at 8.6% per annum
and can deposit Kroner for 3 months at 10% per annum. What is the cost in pounds with a money
market hedge?
Solution: Interest rates for 3 months are 2.15% (8.6%/4) to borrow in pounds and 2.5% (10%/4) to
deposit in kroner. Amount to borrow = amount to deposit = X (1 + 2.5%) = 3500000, X = Kr3414634.
Convert this to will be 3414634/7.5509 = 452215 (spot rate given will always be the bad one).
Company has to borrow 452215 and with 3 months interest will have to repay:
452215 x 1.0215 = 461938.
In 3 months, deposit (Kr3414634 should have increased to Kr3500000) will be taken out to pay the
Danish creditor and company will pay 461938 to the bank.
The choice between forward exchange contract (forward market) and money markets is generally
made on the basis of which method is cheaper, with other factors being of limited significance. We
shall take a look at June 2011 question 4 as example.
Example: ZPS Co, whose home currency is the dollar, took out a fixed-interest peso bank loan several
years ago when peso interest rates were relatively cheap compared to dollar interest rates. Economic
difficulties have now increased peso interest rates while dollar interest rates have remained relatively
stable. ZPS Co must pay interest of 5,000,000 pesos in six months time. The following information is
available.
Per $
Spot rate: pesos
12500 pesos 12582
Six-month forward rate: pesos
12805 pesos 12889
Interest rates that can be used by ZPS Co:
Borrow
Deposit
Peso interest rates:
100% per year
75% per year
Dollar interest rates:
45% per year
35% per year
Calculate whether a forward market hedge or a money market hedge should be used to hedge the
interest payment of 5 million pesos in six months time. Assume that ZPS Co would need to borrow any
cash it uses in hedging exchange rate risk.
Solution: The cost of both methods has to be compared.
Forward market hedge
Cost = 5000000/12.805 = $390472 (always take the rate that is bad).
Money market hedge
The aim is to create a 5000000 peso asset in 6 months time. The amount of pesos required to deposit
is X (1 + 7.5%/2) = 5000000, X = 4819277 pesos. Therefore, the $ to borrow this amount will be
4819277/12.500 = $385542. This is the cost now, we have to include the interest cost for 6 months, ie.
385542 x (1 + 4.5%/2) = $394217.
Comparing the cost, forward market hedge is cheaper by $3745 and should be used to hedge.
71

Asset and liability management


A company which has a long-term foreign investment, for example an overseas subsidiary, can try to
match its foreign assets by a long-term loan in the foreign currency.
Evaluating the foreign currency risk management methods
1. Currency of invoice If the foreign customer wants you to invoice in their currency and you are very
keen to sale to this customer, then you might have to invoice in their currency.
2. Netting - This will really only work effectively when there are many sales and purchases in the
foreign currency. It would not be feasible if the transactions were separated by many months.
3. Matching - For matching to work well, either specific matches are spotted or there have to be many
import and export transactions to give opportunities for matching. Matching would not be feasible if
you received US$2m in November, but didnt have to pay US$1.9m until the following May.
4. Leading and lagging The currency movement may be different from expected.
5. Forward exchange contracts It is cheap and available for many currencies but it is binding and rates
may be unattractive.
6. Money market hedging more time consuming compared to forward exchange contracts.
Foreign currency derivatives
These can be used to hedge foreign currency risk as well. A derivative is a financial instrument whose
value changes in response to the change in an underlying variable. No calculation will be required in
exam.
Currency futures
These can be found in futures market. Currency futures are standardised contracts that oblige the
buyer (seller) to purchase (sell) the specified quantity of foreign currency at a pre-determined price at
the expiration of the contract. Currency futures provide a hedge that theoretically eliminates both
upside (opportunities) and downside (threats) risk by effectively locking the holder into a given
exchange rate, since any gains in the currency futures market are offset by exchange rate losses in the
cash market, and vice versa.
In practice however, movements in the two markets are not perfectly correlated and basis risk exists if
maturities are not perfectly matched. Imperfect hedges can also arise if the standardised size of
currency futures does not match the exchange rate exposure of the hedging company. Initial margin
subsequently required. Futures are generally settled through an offsetting (reversing) trade.
Example: A US exporter is expecting to receive 5m in three months time and that the current
exchange rate is US$/1.24. Assume that this rate is also the price of US$/ futures. The US exporter
will fear that the exchange rate will weaken over the three months, say to US$/1.10 (that is fewer
dollars for a euro). If that happened, then the market price of the future would decline too, to around
1.1. The exporter could arrange to make a compensating profit on buying and selling futures: sell now
at 1.24 and buy later at 1.10. Therefore, any loss made on the main currency transaction is offset by
the profit made on the futures contract.
72

Currency options
This gives holders the right, but not the obligation, to buy (call option) or sell (put option) foreign
currency at specific date at a specific rate. An advantage of currency options over currency futures is
that currency options do not need to be exercised if it is disadvantageous for the holder to do so.
Holders of currency options can take advantage of favourable exchange rate movements in the cash
market and allow their options to lapse. The initial fee (non-returnable premium) paid for the options
will still have been incurred and it depends on the strike price (exercise price of call or put option),
maturity, liquidity in the market and so on.
Example: A UK exporter is expecting to be paid US$1m for a piece of machinery to be delivered in 90
days. If the strengthens against the US$ the UK firm will lose money, as it will receive fewer for the
US$1m. However, if the weakens against the US$, then the UK company will gain additional money.
Say that the current rate is US$/1.40 and that the exporter will get particularly concerned if the rate
moved beyond US$/1.50. The company can buy call options at an exercise price of US$/ = 1.50,
giving it the right to buy at US$1.50/. If the dollar weakens beyond US$/1.50, the company can
exercise the option thereby guaranteeing at least 666,667. If the US$ stays stronger or even
strengthens to, say, US$/1.20, the company can let the option lapse (ignore it) and convert at 1.20, to
give 833,333.
Currency swaps
This is appropriate for hedging exchange rate risk over a longer period of time. It involves exchange of
principal and interest of a loan in one currency for the same in another currency. It begins with an
exchange of principal, although this may be a notional exchange rather than a physical exchange.
During the life of the swap agreement, the counterparties undertake to service each others foreign
currency interest payments. At the end of the swap, the initial exchange of principal is reversed.
4. Hedging techniques for interest rate risk
Interest rate risk can be managed using internal hedging in the form of asset and liability management,
matching and smoothing or using external hedging instruments such as forward rate agreements and
derivatives.
Interest rate risk management methods
Matching and smoothing
Matching (commonly used by banks) is where assets and liabilities with a common interest rate are
matched. For example, one subsidiary could invest in money market at LIBOR and another subsidiary
could borrow through the same market at LIBOR. When LIBOR increases, ones borrowing cost and
anothers returns will increase, therefore matched.
Smoothing is where company keeps a balance between its fixed rate and floating rate borrowing.
Therefore, when the interest rates increase, floating rate loan will be more expensive but this will be
compensated by the less expensive fixed rate loan.

73

Forward rate agreements (FRAs)


This hedges interest rate risk by fixing the interest rate on future borrowing. This protects the
borrower from adverse market interest rates movements to the levels above the rate negotiated for
the FRA. However it is likely to be difficult to obtain FRA for periods of over one year. Something to
note of is the terminology of FRA:
1. 4.00 - 3.80 means that you can fix a borrowing rate at 4.00% (favourable to the bank).
2. A 3 - 6 FRA is one that starts in three months and lasts for three months.
3. A basis point is 0.01%.
Example: It is 30 June. Water Co needs a $10m 6 months fixed rate loan from 1 October. Water Co
wants to hedge using an FRA and the relevant FRA rate is 6% on 30 June. What if the FRA benchmark
rate has moved to 9% later?
Solution: FRA required is 3 - 9. Since the rate increased, Water Co will receive the 3% (9 6) from
bank.
Net payment on loan = 9% x $10m x 6/12 3% x $10m x 6/12 = $300000. Therefore, with FRA the
effective interest rate on loan will be fixed at 6%, in this case Water Co will not lose when the rate
becomes 9%.
Interest rate derivatives
No calculation will be required for these in exam. In considering which instrument to use, consider cost,
flexibility, expectations and ability to benefit from favourable interest rate movements.
Interest rate futures
This is similar to FRAs, except that the terms, amounts and periods are standardised. Because of this,
they cannot always be matched with specific interest rate exposures. With interest rate futures, what
we buy is the entitlement to interest receipts and what we sell is the promise to make interest
payments. Therefore, if a lender buys one 3 month sterling contract, he has the right to receive
interest for three months in pounds.
1. Borrowers will wish to hedge against interest rate rise by selling futures now and buying futures (if
interest rate did rise) on the day that the interest rate is fixed.
2. Lenders will wish to hedge against falling of interest rate by buying futures now and selling futures
(if interest rate did fell) on the date that the actual lending starts.
We can see that borrowers sell futures to hedge against interest rate rises and lenders buy futures to
hedge against interest rate falls.
Normally, futures price is likely to vary with changes in interest rates and outlay to buy futures is much
less than buying a financial instrument itself.
Interest rate swaps
Interest rate swaps are where two parties agree to exchange interest rate commitments. The exchange
could be fixed rate to floating rate or vice versa (plain vanilla or generic swap). For example, party A
agrees to pay the interest on party Bs loan (fixed rate) and party B agrees to pay interest on party As
74

loan (floating rate). If both parties are in different countries, a fixed to floating rate currency swap
(combination of currency and interest rate swap) can be done.
Arranging a swap is cheaper than terminating original loan (termination fee is high) and taking out a
new loan (involve issue costs), therefore companies (normally banks and other types of institution)
swap interest payments with another companies.
Interest rate options
This grants the buyer the right, but not the obligation, to deal at an agreed interest rate (strike rate) at
a future maturity date. On the date of expiry of the option, buyer must decide whether or not to
exercise the right. If a company needs to hedge borrowing, it will purchase a put option to obtain the
right to pay interest at strike rate, so if the interest rate rises at the future date, company will exercise
this option. Similarly, if a company needs to hedge lending, it will purchase a call option.
Interest rate caps, collars and floors
Caps set a ceiling to the interest rate, floor sets a lower limit and collar is the simultaneous purchase of
a cap and sale of floor.
An interest rate cap will compensate the purchaser of the cap if interest rates rise above a
predetermined rate (strike rate) while an interest rate floor will compensate the purchaser if rates fall
below a predetermined rate. Both are also interest rate options.
Interest rate collar is a combination of a purchase of an interest rate cap and a sale of an interest rate
floor to create a range for interest rate fluctuations between the cap and floor strike prices to
minimise the risk of a significant rise in the floating rate. This limits the cost for the company as it
receives a premium for the option (floor) sold.

75

S-ar putea să vă placă și